You are on page 1of 38

A

Sample Questions
A Collective Arrangement by the Chapter Editors, Associate Editors,
and CD-ROM Editor of 2005

COMMUNITY AND PUBLIC HEALTH . . . . . . . . . . . . . . . . . . . .2


MEDICINE . . . . . . . . . . . . . . . . . . . . . . . . . . . . . . . . . . . . . . . . . . . .6
OBSTETRICS AND GYNECOLOGY . . . . . . . . . . . . . . . . . . . . . .11
PEDIATRICS . . . . . . . . . . . . . . . . . . . . . . . . . . . . . . . . . . . . . . . . . .14
PSYCHIATRY . . . . . . . . . . . . . . . . . . . . . . . . . . . . . . . . . . . . . . . . .20
SURGERY . . . . . . . . . . . . . . . . . . . . . . . . . . . . . . . . . . . . . . . . . . . .24
ANSWERS TO SAMPLE QUESTIONS . . . . . . . . . . . . . . . . . . .29
KEY FEATURE QUESTIONS . . . . . . . . . . . . . . . . . . . . . . . . . . . .31
KEY FEATURE SCORING GUIDE . . . . . . . . . . . . . . . . . . . . . . .36

Toronto Notes 2005

Sample Questions 1

2 Sample Questions

Community and Public Health

Toronto Notes 2005

Community and Public Health


1) Which one of the following is not a typical feature of asbestosis?
a) increased risk of cancer
b) pleural thickening and calcification
c) interstitial fibrosis
d) obstructive pattern on pulmonary function tests
e) none of the above
2) The following statements regarding contact dermatitis are true
EXCEPT:
a) phototoxic dermatitis following topical application of
creosote requires UV light
b) photoallergic contact dermatitis requires UV light to be
manifested
c) contact eczema involves a type IV delayed
hypersensitivity reaction
d) contact urticaria or hives is a common form of dermatitis
e) chemical burns by HCl and KOH may result in an irritant
contact dermatitis
3) The following statements regarding noise are true EXCEPT:
a) temporary threshold shift recovers following cessation of
noise exposure
b) permanent threshold shift is characterized by a
progressive pattern of hearing loss
c) most cases of permanent threshold shift are surgically treatable
d) higher frequency noise is more damaging than low frequency
noise
e) none of the above
4) The frequencies most necessary for the understanding of
speech extend from about:
a) 20-20 000 Hz
b) 400-4 000 Hz
c) 250-8 000 Hz
d) 100-5 000 Hz
e) none of the above
5) Lead exposure typically results in:
a) chronic dermatitis
b) resting and intention tremor
c) extensor muscle weakness
d) arrhythmias
e) cerebellar ataxia
6) Which of the following statements concerning the Workers
Compensation Act is true?
a) the worker reserves the right to sue the employer for negligence
b) funding is provided by the provincial government
c) the worker is guaranteed payment from the first day of
injury/illness if it is deemed to be work-related
d) the Workers Compensation Board is an independent, private
agency
e) none of the above
7) Which of following statements regarding radiation is false?
a) natural background radiation accounts for about half of a
typical persons exposure
b) ionizing radiation causes intestinal villi to become denuded
c) exposure to non-ionizing radiation may result in cataracts
d) ionizing radiation results in an increased incidence of
neoplasia such as lung and thyroid
e) none of the above
8) Which statement concerning vibration induced white
finger disease is false?
a) early symptoms include tingling and numbness of the
fingers when at rest
b) swelling of the fingers over knuckles may be an early
feature
c) cold, damp conditions may precipitate symptoms
d) the affected area eventually spreads to involve all fingers
e) none of the above

9) In a cohort study of disease X in people with risk factor


Y versus those who are without risk factor Y, the
following results were obtained:

Y
no Y

no X

80
50

20
50

total
100
100

The relative risk of developing X in Y versus no Y is:


a) 80 x 50 = 4
50 x 20

c) 50 x 80 = 4
20 x 50

b) 50 _ 20 = 0.3
100 100

d) 80/100 = 1.6
50/100

e) it is not possible to calculate risk in a cohort study


10) The attributable risk of factor Y to disease X would be:
a) 80/100 = 1.6
50/100

c) 80 x 50 = 4
50 x 20

b) 80 _ 50 = 0.3
100 100

d) 50 x 20 = 0.25
80 x 50

e) it is not possible to calculate atrributable risk


in a cohort study
11) A group of 50 people are exposed to virus A. Of those 50
people, 9 develop a mild infection, 10 become seriously ill, and 3
die. The attack rate of virus A in the population would be:
a) 22/50
b) 9/50
c) 10/50
d) 19/50
e) 13/50
12) The following indicate the results of screening test Q in
screening for disease Z:
Disease Z
+

Screen Q

40
30

10
120

50
150

70

130

200

The specificity of test Q would be:


a) 40/70
b) 120/130
c) 40/50
d) 120/150
e) 40/130
13) The positive predictive value would be:
a) 40/70
b) 120/130
c) 40/50
d) 120/150
e) 70/200
14) To determine an odds ratio one would have to perform
which of the following studies?
a) a cross sectional/prevalence study
b) a randomized controlled trial
c) a cohort study
d) a case study
e) a case control study

Toronto Notes 2005

Community and Public Health

15) Examples of secondary prevention would include all of the


following EXCEPT:
a) Pap smear for cervical cancer
b) chemoprophylaxis in a recent TB converter
c) proctoscopy for rectal cancer
d) immunization for Haemophilus influenzae B
e) mammography for breast cancer
16) Alpha error is:
a) the probability of declaring a difference to be absent
when it in fact is present
b) the probability of declaring a difference to be present
when it is not
c) the probability of declaring a difference to be absent
when it is indeed absent
d) the probability of declaring a difference to be present
when it does exist

Sample Questions 3

23) Which of the following is the most important justification for


population screening programs for a specific disease?
a) early detection of the disease of interest is achieved
b) the specificity of the screening test is high
c) the natural history of the disease is favourably altered by
early detection
d) effective treatment is available
e) the screening technology is available
24) Regarding the regulation of health professionals, provincial
colleges of physicians and surgeons:
a) have the advancement of the public interest as their
primary goal
b) protect the public from incompetent or unfit MDs
c) act as licensing bodies for MDs
d) do not advance the professional and political interests of MDs
e) all of the above

17) Which one of the following descriptors of a diagnostic test is


influenced by the prevalence of the disease being tested for:
a) specificity
b) sensitivity
c) accuracy
d) positive predictive value
e) reliability

25) Active immunization was important in control of each of the


following childhood communicable diseases EXCEPT:
a) diphtheria
b) polio
c) measles
d) scarlet fever
e) pertussis

18) Which of the following statements regarding the measurement


of health and disease in a population is true?
a) a rate is the number of times an event has occured
during a certain time interval divided by the number of
persons at risk during the same interval
b) when calculating a ratio, the numerator is a portion of
the denominator
c) a ratio is the number of times an event has occurred
during a certain time interval multiplied by the number
of times an event has occurred during the same interval
d) when calculating a rate, the numerator is not a portion of
the denominator
e) none of the above

26) All of the following statements are true EXCEPT:


a) one indirect measure of a populations health status is the
percentage of low birth weight neonates
b) accidents are the largest cause of potential years of life lost
in Canada
c) the Canadian population is steadily undergoing
rectangularization of mortality
d) morbidity is defined as all health outcomes excluding death
e) the neonatal mortality rate is the number of infant deaths
divided by the number of live births multiplied by 1000

19) In 1990, which country spent the least on health care as a


percentage of GDP (gross domestic product):
a) Canada
b) France
c) Sweden
d) UK
e) USA
20) The component of Canada's health care system that
receives the highest percentage of the health care budget is:
a) hospitals
b) physician fees
c) drug benefit plans
d) laboratory services
e) administration
21) The British North America Act (BNA):
a) was mainly concerned with health care legislation
b) granted exclusive powers over health care to the
provinces, including powers over marine hospitals
c) replaced the Canada Health Act
d) applied to British Columbia only
e) none of the above
22) Regarding health expenditure and health outcomes:
a) the U.S. has the greatest health expenditure and the
lowest infant mortality rates
b) there is a positive association between national
expenditure on health and GDP
c) increased national health expenditure always increases
health status of a country
d) all of the above
e) none of the above

27) All of the following statements are true EXCEPT:


a) the data collected on a death certificate is uniform and in
conformity with WHO guidelines
b) Section 7 of the Coroners Act states that the coroners office
must be notified if a patient dies after some mishap such
as leaving an instrument in the body at surgery
c) diseases which must be reported to the local medical
officer of health include AIDS, food poisoning, influenza
and gonorrhea
d) a tuberculin reaction greater than 5 mm is considered
positive in all individuals
e) all of the above
28) In describing the leading causes of death in Canada, two very
different lists emerge, depending on whether proportional
mortality rates or person-years of life lost (PYLL) are used. This
is because:
a) one measure uses a calendar year and the other a fiscal
year to calculate annual experience
b) one measure includes morbidity as well as mortality
experience
c) both rates exclude deaths occurring over the age of 70
d) different definitions of cause of death are used
e) one measure gives greater weight to deaths occurring in
younger age groups
29) Differentiation between a point-source epidemic and a
progressive (propagated) epidemic is made by:
a) considering the characteristics of the infectious agent
b) determining the level of immunity in the community
c) determining the number of persons infected and
calculating the attack rate
d) plotting the distribution of cases by time onset
e) none of the above

4 Sample Questions

Community and Public Health

30) The occurrence of an illness at a rate of above that expected is


called:
a) hyperendemic
b) epidemic
c) endemic
d) enzootic
e) pandemic
31) Each of the following statements applies to case control
studies EXCEPT:
a) starts with disease
b) suitable for rare diseases
c) relatively inexpensive
d) prolonged follow-up required
e) there may be a problem in selecting and matching controls
32) A clinician who has been examining the patterns of mortality in
your community says that the rates for heart disease and lung
cancer are higher in this community than in an adjacent
community. Which of the following questions should you ask first?
a) how did the clinician choose the comparison community?
b) have the rates been standardized for age?
c) are tobacco sales significantly different in the two communities?
d) are the facilities to treat these diseases comparable in the
two areas?
e) are the numbers of deaths comparable in each area?
33) The purpose of randomization is to:
a) make sure that there are equal numbers of men and women
in test and control groups
b) increase the chances of getting a statistically significant difference
c) ensure that the numbers of cases and controls are equal
d) limit bias
e) all of the above
34) Which of the following types of studies usually provides only a
measure of prevalence?
a) descriptive
b) cross-sectional
c) randomized controlled trial
d) cohort
e) none of the above
35) The major advantage of cohort studies over case-control
studies is that:
a) they take less time and are less costly
b) they can utilize a more representative population
c) it is easier to obtain controls who are not exposed to the factor
d) they permit estimation of risk of disease in those exposed
to the factor
e) they can be done on a double-blind basis
36) All of the following statements concerning occupational
health are true EXCEPT:
a) disorders of reproduction are among the top 10
work-related diseases and injuries
b) most workers are covered by both federal and provincial
legislation with respect to workplace health and safety
c) skin problems and hearing problems together are
responsible for half of WCB claims
d) a complete occupational medical history includes
investigation of the temporal relationship between
symptoms and exposure
37) Which of the following statements concerning exposure to solvents
in the workplace is true?
a) each solvent compound has a specific antidote that can be
used to treat exposure
b) a prominent symptom of solvent exposure is memory loss
c) some solvents can cause skin dryness and loss of
subcutaneous adipose tissue
d) solvents do not affect the bone marrow
e) all of the above

Toronto Notes 2005

38) In 1981, the crude birth rate in Ontario was approximately


14 per 1000 and the crude death rate was 7 per 1000. The
estimated rate of net migration was 1 per 1000. The growth
rate of the province, per 1000 population was:
a) 6
b) 7
c) 8
d) 20
e) 22
39) All of the following statements about environmental
health are true EXCEPT:
a) levels of toxic agents measured in the environment may not
reflect internal organ levels
b) the federal government monitors the quality and types of
industrial emissions and toxic waste disposal
c) sick building syndrome is associated with Pontiac fever and
Legionnaires disease
d) all humans have detectable levels of PCBs
e) none of the above is true
40) The effectiveness of a preventative measure is assessed in terms of:
a) the effect in people to whom the measure is offered
b) the effect in people who comply with the measure
c) availability with the optimal use of resources
d) the cost in dollars versus the benefits in improved health status
e) all of the above
41) All of the following statements about the Canada Health
Act (1984) are true EXCEPT:
a) it did not define all medically necessary hospital and
physician services
b) the CHA replaced the Hospital Insurance and Diagnostic
Services Act of 1957
c) the CHA banned all forms of extra billing
d) according to the CHA, provinces must meet all the terms
and conditions of Medicare to qualify for federal transfer
payments
e) none of the above
42) Each of the following is an example of primary prevention EXCEPT:
a) genetic counselling of parents with one retarded child
b) nutritional supplements in pregnancy
c) immunization against tetanus
d) chemoprophylaxis in a recent tuberculin converter
e) speed limits on highways
43) The classical epidemiological triad of disease causation consists of
factors which fall into which of the following categories:
a) host, reservoir, environment
b) host, vector, environment
c) host, agent, environment
d) reservoir, agent, vector
e) host, age, environment
44) Of the five items listed below, the one which provides the strongest
evidence for causality in an observed association between exposure
and disease is:
a) a large attributable risk
b) a large relative risk
c) a small p-value
d) a positive result from a cohort study
e) a case report
45) The difference between a common point source outbreak and
a propagated outbreak of illness is that:
a) all cases in a common point source outbreak occur within
one incubation period of the exposure
b) the attack rates in propagated outbreaks are higher
c) person-to-person transmission is a feature of common
source outbreaks
d) case fatality rates in common source outbreaks are generally
higher
e) the source of infection in propagated outbreaks is more
easily contained than in common source outbreaks

Toronto Notes 2005

Community and Public Health

46) Which of the following are strategies for control of disease:


a) population immunization
b) contact tracing to offer treatment to all who could be infected
c) monitoring increases in the population of certain disease vectors
d) having physicians report curable, potentially serious diseases
e) all of the above are true
47) During a clinical trial, the difference in the success rates of two
drugs was not statistically significant. This means that:
a) there is no difference in drug effectiveness
b) there is a sizable probability that the demonstrated difference in
the drugs effectiveness could occur due to chance alone
c) the demonstrated difference in the drugs effectiveness is
too small to be clinically meaningful
d) the two samples of patients on which the drugs were tested
came from the same population
e) none of the above are true
48) The incidence of a particular disease is greater in men than in
women, but the prevalence shows no sex difference. The most
probable explanation is that:
a) the mortality rate is greater in women
b) the case fatality rate is higher in women
c) the duration of the disease is longer in women
d) women receive less adequate medical care for the disease
e) this diagnosis is more often missed in women
49) All of the following statements about statistical tests are true
EXCEPT:
a) linear regression is used to describe the relationship
between two continuous variables
b) a confidence interval is a range of values giving information
about the precision of a measurement
c) ANOVA tables are used to make comparisons among the means
of 3 or more groups simultaneously
d) in a normal distribution, the mean, median and mode are equal
e) the chi-square test evaluates the statistical significance of 2
or more percentages of categorical outcomes

Sample Questions 5

50) All of the following are responsibilities of local public health


units in Canada EXCEPT:
a) communicable disease control
b) health education
c) investigation of sudden death
d) immunization
e) health promotion
51) Who is ultimately responsible for the quality of care in a hospital?
a) chief executive officer
b) board of trustees
c) medical director
d) chief of staff
e) attending physicians
52) Which of the following is not one of the 5 Terms and
Conditions of Medicare?
a) portability
b) flexibility
c) universality
d) comprehensive coverage
e) accessibility
53) You are given this data pertaining to an outbreak of diarrhea
in a daycare:
Age Number of Children
Number of Diarrhea
1
20
17
2
19
15
3
39
13
4
39
4
5
38
5
6+
18
1
What is the attack rate of this illness?
a) 25%
d) 40%
b) 30%
e) 50%
c) 32%

6 Sample Questions

Medicine

Toronto Notes 2005

Medicine
1) Which of the following is a feature of secretory diarrhea?
a) small stool volume (< 1L/day)
b) increased stool osmotic gap
c) persistent diarrhea despite fasting
d) blood and/or pus in stools
e) malodorous, often floating stools

9) Hemolytic anemia is characterized by all of the following EXCEPT:


a) increased LDH
b) increased reticulocytosis
c) increased unconjugated bilirubin
d) increased haptoglobin
e) lead poisoning

2) Each of the following are risk factors for colon cancer EXCEPT:
a) low fiber diet
b) severe diverticular disease
c) familial adenomatous polyposis
d) ulcerative colitis
e) high fat diet

10) A patient complains of a non-tender mass over the thyroid region


on the left side of her neck. Concerned about a thyroid disorder,
you order the appropriate investigations. The results are as
follows:
TSH: 6.0
Free T4: 20.2
Thyroid antibodies: none
RAIU: No hot spots seen

3) Intestinal complications more common to Crohns disease than


ulcerative colitis include each of the following EXCEPT:
a) fistula formation
b) perianal disease
c) intestinal obstruction
d) toxic megacolon
e) post-surgical recurrence
4) A 21 year-old bisexual man has a 4 week history of intermittent
diarrhea, urethral discharge, and pain in the right knee and left
second toe. He has several oral ulcers, a clear urethral discharge, a
scaly papular rash on palms and soles, onycholysis, sausage-like
swelling of the left second toe, and heat and swelling of the right
knee. The results of Gram stains and cultures of urethral discharge
are negative. Rheumatoid factor is not present. The most likely
diagnosis is:
a) Reiters syndrome
b) gonococcal arthritis
c) Behcet disease
d) acquired immune deficiency syndrome
e) psoriatic arthritis
5) A 53 year-old presents to your office with pain and stiffness in
both hands and knees of 6 months duration. Which of the
following findings on your physical examination may help with a
diagnosis:
a) joint tenderness/effusions
b) maculopapular rash
c) iridocyclitis
d) hepatosplenomegaly
e) all of the above
6) Radiographic features of osteoarthritis of the knee include
which of the following:
a) marginal erosions
b) juxta-articular osteopenia (demineralization)
c) loss of articular cartilage with narrowing of the radiologic
joint space
d) osteonecrosis (avascular necrosis) of the medial femoral
condyle
e) high riding patella (patella alta)
7) An elevated level of hemoglobin A2 in a patient with mild
microcytic anemia suggests the diagnosis of:
a) alpha-thalassemia
b) sickle trait
c) beta-thalassemia
d) hereditary spherocytosis
e) hereditary persistence of fetal hemoglobin
8) It is unlikely to see macrocytosis in a patient with anemia in which
of the following?
a) reticulocytosis
b) vitamin B12 deficiency
c) folate deficiency
d) myelodysplastic syndrome
e) sideroblastic anemia

The next investigation(s) you choose to do are:


a) watch and wait for 3-6 months
b) FNA
c) surgical biopsy
d) trial of L-thyroxine therapy for 6 months
e) none of the above
11) An 8 year-old boy is brought to the office because his mother is
concerned he is entering puberty already. You examine him and
note the beginnings of facial hair, axillary hair and Tanner stage 2
external genitalia. Choose the set of investigations you initially
want to do:
a) CBC, lytes, testosterone, bone age, CT head
b) FSH, LH, testosterone, lytes, bone age, DHEA-S
c) FSH, LH, testosterone, cortisol, DHEA-S, 11-OH progesterone,
bone age
d) lytes, testosterone, DHEA-S, 17-OH progesterone, cortisol, bone
age
12) The subendothelium is the most vulnerable segment of the heart
from an ischemic standpoint. The major reason for this is:
a) the highest oxygen utilization is in the subendocardium
b) coronary flow to the subendocardium occurs almost completely
during diastole whereas other regions receive some flow
during systole as well
c) the subendocardium has a diminished aerobic capacity
d) there is less potential for collateralization to the
subendocardium
e) the ratio of capillary to myocyte is less in the subendocardium
than in other regions
13) 18 year-old female with initial onset of pruritic rash characterized
by excoriations, scaling and crusting and distributed on the
extremities, neck and eyelids. Past medical history is significant for
asthma and hayfever. The most likely diagnosis is:
a) scabies
b) atopic dermatitis
c) contact dermatitis
d) shingles
e) dyshydrotic eczema
14) 8 year-old black male comes in with an asymptomatic
erythematous eruption characterized by oval patches with collarette
scaling. It is distributed as a "Christmas tree" pattern on the back.
Father states that there was originally one lesion on the abdomen a
few weeks prior. What is the most likely diagnosis:
a) pityriasis rosea
b) tinea corporis
c) lichen planus
d) psoriasis
e) none of the above

Toronto Notes 2005

Medicine

15) 30 year-old female comes in with a soft smooth erythematous


nodule on her lower lip. She states that a few weeks prior she had
some chapped lips with occasional bleeding. Now, the lips have
healed but this lesion arose suddenly in its place. It is occasionally
tender on pressure. The most likely diagnosis is:
a) HSV1
b) cherry hemangioma
c) pyogenic granuloma
d) dermal nevus
e) none of the above
16) A 65 year-old male with back pain, nephrotic syndrome and
anemiapresent to the ER. Ultrasound shows normal kidney size.
His creatinine is 500. Which diagnosis best fits the scenario?
a) polycystic kidney disease
b) chronic GN
c) multiple myeloma
d) diabetic nephropathy
e) analgesic abuse
17) Which of the following are indications for dialysis in ARF?
a) severe alkalosis unresponsive to medical therapy.
b) severe acidosis unresponsive to medical therapy.
c) severe hypokalemia unresponsive to medical therapy.
d) severe hypercalcemia unresponsive to medical therapy.
e) b and d
18) Which of the following is true with respect to diabetes and kidney
disease?
a) primarily affects the tubules.
b) earliest sign is decreased GFR.
c) microalbuminuria is a late sign of DM nephropathy.
d) threshold for dialysis is same as other CRF patients.
e) BP control slows progression of DM nephropathy.
19) The various species of Campylobacter can cause diseases ranging
from acute enteritis to bacteremia. Which of the following modes of
transmission does NOT apply to Campylobacter?
a) contact with infected animals
b) contaminated food and water
c) improperly cooked poultry
d) aerolized droplets
e) person to person spread via fecal-oral route
20) Which of the following is NOT a common infectious cause of acute
diarrhea?
a) Escherichia coli
b) Shigella
c) Norwalk virus
d) Vibrio cholerae
e) Helicobacter pylori
21) Impaired coronary flow reserve is associated with each of the
following conditions EXCEPT:
a) severe aortic stenosis
b) severe systemic hypertension with left ventricular hypertrophy
c) severe mitral stenosis in the presence of atrial fibrillation
d) a totally occluded coronary artery but with excellent collateral
supply from the contralateral (i.e. opposite) coronary artery
e) an isolated 30% diameter stenosis of a coronary artery
22) Which of the following pulmonary function tests most
reliably discriminates pure chronic bronchitis from
emphysema?
a) total lung capacity
b) functional residual capacity
c) residual volume
d) single breath diffusing capacity
e) flow at 50% vital capacity
23) Which of the following would NOT be part of your plan for the
treatment of acute ventricular fibrillation?
a) electrical defibrillation
b) lidocaine
c) epinephrine
d) bretylium
e) manganese

Sample Questions 7

24) Which of the following is not an aggravating factor of congestive


heart failure?
a) hypertension
b) thyrotoxicosis
c) alcohol
d) inactivity
e) arrhythmia
25) In the course of DKA, serum potassium levels:
a) remain unaffected
b) can appear normal but total body potassium may actually be
low
c) can appear normal but total body potassium may actually be
high
d) will naturally be corrected by insulin administration
e) none of the above
26) A 2 month-old boy has a Ca of 1.80 (corrected) after an assessment
for FTT. Mother informs you she has been consistently
breastfeeding without trouble as corroborated by a visiting nurse.
What is at the top of your differential?
a) malabsorption
b) pseudohypoparathyroidism
c) mother didnt supplement with DiVisol (Vit D supplement)
d) DiGeorge syndrome
27) A virus that is not inactivated by mild detergents that solubilize
phospholipid membranes is:
a) poliovirus
b) variola virus
c) cowpox virus
d) vaccina virus
28) All the following are true statements about viruses EXCEPT:
a) they are obligate intracellular parasites
b) they are filterable agents
c) they are simply organized
d) they are devoid of enzymes
e) they may contain double stranded DNA
29) A viral genome that does not replicate in the cytoplasm of the
infected cell is:
a) poliovirus
b) rabies virus
c) cytomegalovirus
d) rubella virus
e) mumps virus
30) Which of the following is true about congenital heart block in
neonatal lupus erythematosus?
a) it is associated with maternal anti-Ku autoantibodies
b) it is transient
c) the majority of patients will require a pacemaker
d) there is no increased risk of connective tissue disease in
adulthood
e) the risk of mortality is small
31) Which of the following is true about serologic testing in SLE?
a) Aapositive ANA is specific for SLE
b) ds-DNA level correlates with disease activity in SLE
c) anti-histone antibodies are seldom positive in non-drug induced
SLE
d) the majority of patients with SLE have anti-Sm antibodies
e) anti-Ro antibody is specific for SLE
32) The treatment of choice for thrombotic events in the
antiphospholipid antibody syndrome is:
a) intravenous steroids
b) high-dose oral steroids with a rapid taper
c) penicillamine
d) aspirin
e) warfarin

8 Sample Questions

Medicine

33) Highly infective chronic hepatitis B is suggested by:


a) elevated liver enzymes, HBeAg+, anti-HBc IgM+
b) normal liver enzymes, HBeAg+, anti-HBc IgG+
c) normal liver enzymes, HBeAg-, anti-HBc IgG+
d) elevated liver enzymes, HBeAg+, anti-HBc IgG+
34) Which is more often associated with hospital acquired pneumonia
than community acquired pneumonia?
a) Streptococcus pneumoniae
b) Hemophilus influenza
c) Legionella
d) Chlamydia pneumoniae
e) Mycoplasma pneumoniae
35) Which of the following is least likely to contribute to myeloma?
a) hypercalcemia
b) amyloidosis
c) infiltration of the kidney by myeloma cells
d) hyperuricemia
e) intratubular light chain deposition
36) All of the following are vitamin-K dependent proteins EXCEPT:
a) protein C
b) antithrombin III
c) factor IX
d) factor II
e) factor VII
37) An 11 year-old male comes in with erythematous pustules,
inflamed nodules and cysts with some scaring distributed on the
face predominantly. Diagnosis of acne vulgaris was given.
Topical erythromycin was used for 2 weeks, several months ago,
with no response. What treatment would you prescribe now?
a) accutane immediately
b) topical tretinoin
c) topical benzoyl peroxide
d) topical antibiotic other than erythromycin
e) oral antibiotic
f) oral antibiotic and topical tretinoin
38) A 40 year-old woman develops recurrent papules and pustules in
a symmetrical pattern on her cheeks, nose, chin and forehead. She
blushes easily, especially when consuming hot liquids, alcohol, or
spicy foods. The most likely diagnosis is:
a) acne vulgaris
b) perioral dermatitis
c) acne rosacea
d) seborrheic dermatitis
e) carcinoid syndrome
39) A 27 year-old man is brought into the ER after a bicycling accident.
A car door suddenly opened in front of him, of which he smashed
into and was thrown 15 feet. On examination, he is drowsy and
confused. He opens his eyes when his name is called. He mumbles
words that you understand but the sentences do not make sense.
He moves all four limbs but does not respond to any commands.
He is able to pull both hands away when pinched and squirms
when his sternum is rubbed, making no effort to stop you. What is
his Glasgow COMA Scale score?
a) 10
b) 11
c) 9
d) 8
e) 7
40) A 74 year-old, right-handed man presents with a past medical
history of hypertension and dyslipidemia for 30 years. He is a
retired banker who recently has had trouble calculating his
restaurant bill. He also notices that his writing has deteriorated.
On physical exam, he has difficulty naming his fingers and is
confused with distinguishing left from right. The lesion is most
likely in which part of the brain?
a) right parietal
b) left parietal
c) left temporal
d) right temporal
e) frontal

Toronto Notes 2005

41) Which of the following pair of CNS lesions and corresponding


visual field defects is incorrect?
a) temporal lobe tumour superior quadrantanopia
b) frontal lobe tumour altitudinal field defect
c) pituitary tumour bitemporal hemianopsia
d) occipital lobe tumour homonomous hemianopsia
e) Multiple Sclerosis central scotoma
42) The triple bolus test of pituitary function works by a rapid
succession of IV constituents as follows:
a) insulin hypoglycemia mediated rise in GH and ACTH
GHRH rise in LH and FSH
TRH rise in TSH and PRL
b) CRH rise in GH and ACTH
GHRH rise in LH and FSH
TRH rise in TSH and PRL
c) estrogen rise in LH, drop in FSH and PRL
insulin rise in GH and ACTH
TRH TSH
d) cortrosyn rise in GH and ACTH
GHRH rise in LH and FSH
TRH rise in TSH and PRL
43) A 58 year-old man with a past history of a parathyroidectomy for
primary hyperparathyroidism is now in your office complaining
of headaches worse in the AM (made worse by a small MVA he
credits to a loss of peripheral vision). You plan to:
a) send to the Emergency Department for an immediate CT head
b) check his calcium to ensure theres no remaining parathyroid
tissue
c) check for a pheochromocytoma (which you know causes
H/As) because you are concerned he has MEN I syndrome
d) check for a homonymous hemianopia because you are
worried about a pituitary tumor
e) check for a bitemporal hemianopia because you are worried
about a pituitary tumor
44) A 30 year-old patient with asthma complains of daily wheezing
and occasional waking at night with cough and chest tightness for
three weeks. His usual medication is salbutamol two puffs
tid-qid. The next step in management is:
a) add long-term theophylline
b) increase salbutamol to two puffs q4h
c) add ipratropium bromide two puffs qid
d) add beclomethasone two puffs qid
e) discontinue salbutamol and begin prednisone 50 mg od and
taper over 2 weeks
45) Which is not a feature of asbestosis?
a) Increased risk of cancer.
b) Pleural thickening and effusion.
c) Interstitial fibrosis.
d) Obstructive pattern on pulmonary function tests.
e) All of the above are features of asbestosis.
46) A 63 year-old woman develops intermittent dizziness.
Examination discloses diminished corneal light reflex and mild
hearing loss in theright ear. The most likely diagnosis is:
a) cerebellopontine angle tumour
b) benign paroxysmal positional vertigo
c) lateral medullary syndrome
d) Mnire disease
e) none of the above
47) A 25 year-old man is admitted with a history suggesting seizures.
Which of the following would not support this diagnosis?
a) urinary incontinence
b) the sound of voices preceding events
c) drowsiness and weakness following the event
d) rarely occur when recumbent
e) none of the above
48) Which of the following would not be expected in a right-sided
Brown-Squard syndrome?
a) right-sided hemi-paresis
b) right-sided decreased proprioception
c) left-sided decreased sensitivity to pinprick
d) left-sided decreased vibration sense
e) none of the above

Toronto Notes 2005

Medicine

49) Endocarditis in an I.V. drug user:


a) is equally prevalent to that of the normal population
b) is commonly located in the mitral valve
c) is typically caused by S. pneumoniae
d) is typically found on the tricuspid valve
e) is typically found on the aortic valve, producing a systolic
ejection murmur
50) A 55 year-old man with a history compatible with chronic
bronchitis presents to your office with shortness of breath on exam.
In the history, all of the following would be anticipated EXCEPT:
a) A 20-year history of smoking
b) worsening of symptoms with exposure to smog
c) worsening of symptoms with acute respiratory infections
d) recurrent episodes of pleurisy
e) increased incidence of chronic respiratory disease in family
members
51) Which of the following are not consistent with primary
(spontaneous) bacterial peritonitis?
a) abdominal discomfort and fever
b) ascitic fluid neutrophil count of> 250x106 cells/L
c) ascitic fluid WBC count of >500x106 cells /L
d) multiple organisms on culture and sensitivity of ascitic fluid
52) Which finding is NOT frequently found in Chronic Myelogenous
Leukemia (CML)?
a) elevated WBCs
b) elevated vitamin B12 level
c) elevated LDH
d) translocation between chromosomes 9 and 14
e) increased uric acid level
53) Schistocytes on blood film examination are UNLIKELY to be seen
in which of the following:
a) thrombotic thrombocytopenia purpura (TTP)
b) thalassemia
c) vasculitis
d) disseminated intravascular coagulation (DIC)
e) glomerulonephritis
54) All of the following can INHIBIT the absorption of ingested
non-heme iron EXCEPT:
a) alcohol
b) achlorhydria
c) phosphate (i.e. as found in milk)
d) phytates (i.e. as found in cereals)
e) antacids
55) Which of the following is true of Myasthenia Gravis?
a) in patients older than 60, thymic hyperplasia is a common
etiology
b) often associated with thyroid disease
c) antibodies that are produced against acetylcholinesterase
d) associated with small cell lung carcinoma
56) Which of the following is true with respect to proteinuria?
a) all proteinuria is secondary to glomerular disease
> 2 g/24 h = nephrotic syndrome
b) is always abnormal and indicative of serious renal disease
c) it may be normal for an individual to have <150 mg per day of
proteinuria
d) if a patient has 1.5 g of protein in 24 h they must have
tubular-interstitial disease.
57) In which of the following conditions would one not expect a
Trans-Tubular Potassium Gradient greater than 4?
a) primary hypoaldosteronism
b) acute vomiting
c) renin-secreting tumour
d) unilateral renal artery stenosis
e) Gordon syndrome
58) The most significant cause of morbidity in the elderly:
a) arthritis
b) dementia
c) heart disease
d) stroke
e) hearing impairment

Sample Questions 9

59) Which of the following is NOT an age-related change?


a) impaired myocardial diastolic dysfunction
b) increased gastric acid secretion
c) decreased drug clearance
d) increased nocturnal sodium and fluid excretion
e) decreased baroreflex sensitivity
60) Regarding the elderly patient, which of the following apply?
a) vague symptoms
b) atypical presentations
c) loss of function
d) polypharmacy
e) all of the above
61) Which of the following is not associated with thyroid disease?
a) dermatitis herpetiformis
b) urticaria
c) porphyria cutanea tarda
d) vitiligo
e) alopecia areata
62) All of the following are treatments for non-scarring alopecia except:
a) spironolactone
b) minoxidil
c) hair transplantation
d) intralesional triamcinalone
e) finasteride
63) A 70 year-old woman presents with acute knee arthritis.
Radiographs show meniscal calcification (chondrocalcinosis).
Analysis of the synovial fluid reveals weakly positive
birefringent rhomboid-shaped crystals. The crystals are
most likely:
a) monosodium urate
b) calcium hydroxyapatite
c) cholesterol
d) calcium pyrophosphate dihydrate
e) dicalcium phosphate dihydrate (Brushite)
64) Entamoeba histolytica is transmitted to humans by:
a) ingestion of infective eggs
b) ingestion of cysts
c) ingestion of animal tissue that contains the larva
d) penetration of the skin by infective larva
e) ingestion of adult form
65) Strongyloides spp. is transmitted to humans by:
a) ingestion of infective eggs
b) ingestion of cysts
c) ingestion of animal tissue that contains the larva
d) penetration of the skin by infective larva
e) ingestion of adult form
66) The biosynthesis of fungal ergosterol is inhibited by:
a) amphotericin B
b) griseofulvin
c) flucytosine
d) nystatin
e) ketoconazole
67) A 55 year-old woman with asthma is on systemic steroids for one
year. She develops a recent right-sided pleural effusion. She feels
unwell and tires easily. Aspiration reveals a turbid fluid, a high
lymphocyte count, high LDH, low glucose, and a pH of 7.4. The
most compatible diagnosis is:
a) pulmonary embolism
b) empyema
c) tuberculosis
d) subphrenic abscess
e) pancreatitis

10 Sample Questions

Medicine

68) Diagnosis of acute symptomatic pulmonary embolism can be


excluded when which of the following is normal?
a) chest x-ray
b) ventilation-perfusion lung scan
c) bilateral leg venograms
d) PaO2 and A-a O2 gradient
e) CT scan of the pulmonary arteries
69) In acute pyelonephritis, which of the following is most commonly
associated with bacteremic spread from a distant focus?
a) Escherichia coli
b) Proteus sp.
c) Staphylococcus aureus
d) Serratia sp.
e) Enterococcus sp.
70) A patient presents with a decreased level of consciousness and
visual difficulties. Bloodwork reveals an anion gap of 22 and an
osmolar gap of 24. Which of the following is most likely
responsible?
a) ethanol
b) salicylates
c) renal tubular acidosis type I
d) methanol
e) diabetic ketoacidosis
71) In the treatment of Type I Diabetes, which of the following is true?
a) sulfonylureas are useful as an adjunctive therapy to insulin
b) most patients are adequately controlled with one type of
insulin (non-mixed) only
c) once diagnosed with Type I, patients must immediately be
assessed for retinopathy
d) during periods of illness or infection, patients may require
additional insulin
e) the most common initial presentation is visual disturbance
72) Which of the following is associated with thyroid disease?
a) neurofibromatosis
b) vitiligo
c) erythema nodosum
d) pemphigus vulgaris
e) icthyosis vulgaris
73) Each of the following is a correct statement about COPD EXCEPT:
a) the type of emphysema associated with smoking is usually
centriacinar
b) clubbing is not a clinical feature
c) long-term oral steroids should be avoided
d) smoking cessation does not lead to improvement of
pulmonary function
e) the aim of supplemental O2 therapy is to provide relief of
shortness of breath

Toronto Notes 2005

74) Nitroglycerin administered sublingually may contribute to the


relief of myocardial ischemic pain by each of the following
mechanisms EXCEPT:
a) coronary vasodilation
b) decreased venous pooling resulting in increased cardiac
preload
c) reduced systemic vascular resistance
d) reduced ventricular volume
75) At what CD4 count are HIV patients at increased risk of
developing PCP?
a) CD4 count >500x106
b) CD4 count 200-499x106
c) CD4 count <200x106
d) CD4 count < 500x106
e) none of the above
76) An 80 year-old female with a 25% reduction in her GFR requires
which of the following adjustments for a drug that is
predominantly cleared by the kidneys?
1. 25% decrease in individual doses
2. 25% increase in dosing intervals
3. increase overall fluid intake
4. decrease length of therapy by 25%
a) all of the above
b) 1 and 3
c) 2 and 4
d) 2,3, and 4
e) 1 and 2
77) Consequences of immobility include which ONE of the following:
a) diarrhea
b) maintenance of muscle mass
c) urinary retention
d) pneumonia
e) hastened wound healing
78) Which of the following is FALSE in terms of falls in the elderly?
a) they are the most common cause of mortality due to injury
b) environment plays a significant role
c) fractures most commonly involve the humerus
d) age-related sensory changes make the elderly more
susceptible
e) fear of falling contributes to self-protection immobility

Toronto Notes 2005

Obstetrics and Gynecology

Sample Questions 11

Obstetrics and Gynecology


1) A 26 year-old primigravida presents at 40 weeks in active labour
with contractions every 2 minutes. She is diagnosed as having a
transverse lie with the back up. Which of the following would be
the most appropriate next step?
a) start isoxuprine (Vasodilan)
b) perform an external version
c) prepare for an immediate C-section
d) rupture membranes and then perform an internal version
e) none of the above
2) A 25 year-old primigravida at 34 weeks gestation is thought to be
small for dates by her physician and is sent for a sonographic
evaluation. The ultrasound shows the biparietal diameter to be
appropriate for 34 weeks gestation. The abdominal circumference
is appropriate for 30 weeks gestation. The head:abdominal
circumference ratio is < 1. The estimated fetal weight is <10th
percentile for 34 weeks gestation. The amniotic fluid is decreased.
Which of the following is the most likely diagnosis?
a) symmetrical IUGR
b) asymmetrical IUGR
c) congenital anomaly
d) congenital infection
e) unknown gestational age
3) A 32 year-old G5P4 presents with an 8 week history of amenorrhea
andsuggestive symptoms of pregnancy. Physical examination
reveals an irregular, enlarged uterus of 16 weeks size. Ultrasound
confirms thepresence of an 8 week viable pregnancy and a
multiple fibroid uterus. The correct management for this patient is:
a) termination of the pregnancy with elective myomectomy
2 months later
b) termination of the pregnancy with concomitant myomectomy
c) prudent observation with elective C-section at term
d) prudent observation anticipating probable vaginal delivery
e) myomectomy and follow pregnancy in usual way
4) An infant is born. At one minute the heart rate is 120 per minute,
respiratory effort is a good strong cry, muscle tone is active, reflex
irritability is absent, colour is pink with blue extremities. What is
the one minute Apgar score?
a) 5
b) 6
c) 7
d) 8
e) 9
5) Which of the following is the easiest and most reliable way of
detecting a retained succenturiate placental lobe?
a) palpation of the uterus
b) inspection of the maternal side of the placenta
c) visualization of the cervix on speculum exam
d) inspection of the fetal side of the placenta
e) pelvic ultrasound
6) Choose the correct statement regarding operative obstetrics:
a) a first degree tear involves skin and vaginal mucosa as well as
underlying fascia and muscle
b) delivery aided by vacuum extraction is typically of shorter
duration than if forceps are used
c) one absolute prerequisite for the use of forceps is that the
babys presenting part must be below the ischial spines
d) a third degree tear extends through the anal sphincter
e) none of the above
7) A 36 year-old female presents with a mucopurulent vaginal
discharge. Gram stain of a cervical swab shows gram negative
diploccocci. Which of the following is correct?
a) if untreated this condition will likely resolve spontaneously
b) the most likely diagnosis is Strep group B
c) appropriate treatment is Metronidazole vaginal cream
d) appropriate treatment is a single I.M. dose of 250 mg
of Ceftriaxone
e) Public Health will most likely need to be notified

8) A one year-old female presents with vaginal bleeding. Vaginal


inspection reveals the presence of a multicystic grape-like lesion.
The most likely diagnosis is?
a) sexual abuse
b) DES syndrome
c) sarcoma botyroides
d) clear cell adenocarcinoma
e) exposure to exogenous estrogen
9) A 24 year-old G1P0 is seen in the Emergency Department. Her LMP
was 8 weeks ago. She is experiencing lower abdominal cramping
and heavy vaginal bleeding with clots. Examination reveals a soft
abdomen with mild lower abdominal tenderness. On pelvic exam,
the vagina is filled with blood and clots. The cervical os is opened
and tissue is protruding. The uterus is enlarged to a 6 week size.
Which of the following is the most likely diagnosis?
a) ectopic pregnancy
b) threatened abortion
c) degenerating fibroid
d) placenta previa
e) incomplete abortion
10) A 24 year-old female G1P0 is admitted to hospital at 34 weeks
gestation. The patient describes passing a large volume of clear fluid
per vagina for 36 hours. Examination reveals the following: temp
38.5C, no uterine contractions, vague lower abdominal tenderness,
fetal heart rate 185 bpm. Lab investigations reveal: Hb 120, WBC
19.0, + nitrazine test of vaginal fluid, + ferning of vaginal fluid.
Which of the following is the most likely diagnosis?
a) premature rupture of membranes
b) premature rupture of membranes and chorioamnionitis
c) premature labour
d) acute pyelonephritis
e) acute vaginitis of pregnancy
11) A 24 year-old female presents with abdominal pain. -HCG is
negative. Pelvic ultrasound shows a 5 cm right ovarian cyst. You
would:
a) perform immediate laporotomy
b) perform immediate laproscopy
c) aspirate the cyst under ultrasonographic guidance
d) order a CBC and a CA125
e) expectant management with repeat ultrasound in 8 weeks
12) The pathology report following a therapeutic abortion shows
Arias-Stella phenomenon (decidualization). The most appropriate
next step in the management of this patient would be?
a) repeat a pregnancy test
b) laporoscopy
c) reassure the patient
d) prescribe oral contraception
e) repeat the D&C
13) In a patient whose blood type is O-Rh negative and whose husband
has O-Rh positive blood, Rh immune globulin (Rhogam) should be
given after all the following EXCEPT:
a) artificial rupture of membranes
b) full term delivery of a baby with O-Rh positive blood type
c) amniocentesis
d) spontaneous abortion
e) therapeutic abortion
14) Risk factors for shoulder dystocia include all the following EXCEPT:
a) maternal obesity
b) macrosomia
c) maternal diabetes
d) prolonged second stage of labour
e) outlet forceps delivery

12 Sample Questions

Obstetrics and Gynecology

Toronto Notes 2005

15) Fetal exposure to lithium in the first trimester has been strongly
associated with:
a) alopecia
b) cleft lip and palate
c) congenital heart defects
e) limb defects
d) mental retardation

25) A 55 year-old woman comes to the office to discuss menopause.


You tell her all of the following are a part of this stage EXCEPT:
a) hot flushes
b) vaginal dryness
c) mood swings
d) insomnia
e) decreased libido

16) The commonest type of bleeding encountered with uterine


leiomyomata is:
a) post-coital spotting
b) mid-cycle bleeding
c) hypermenorrhea
d) oligomenorrhea
e) post-coital staining

26) All of the following features have been described in women with
androgen insensitivity syndrome EXCEPT:
a) mammary aplasia
b) female phenotype
c) absence of wolffian duct structures
d) short vagina
e) absence of mullerian duct structures

17) Which of the following statements about estrogen therapy in


postmenopausal women is correct?
a) it is a major risk factor for breast cancer
b) it decreases hepatic triglyceride production
c) it produces hypertension
d) it protects against vertebral compression fractures
e) it directly stimulates the proliferation of vascular endothelium

27) The commonest indication for amniocentesis is:


a) chromosome anomaly of the mother
b) teenage pregnancy
c) advanced maternal age
d) family history of Down syndrome
e) family history of cystic fibrosis

18) Which event is matched with the correct timing?


a) maternal serum screening test: 20-22 weeks
b) post-partum visit: 6 weeks
c) chorionic villus sampling: 4-6 weeks
d) oral glucose challenge test: 14-16 weeks
e) Rhogam to Rh negative women: 12-14 weeks
19) Routine blood work during the first antenatal visit include all of
the following EXCEPT:
a) CBC
b) HIV
c) rubella titre
d) blood group and type
e) Rh antibodies
20) All of the following are cardiovascular adaptations to pregnancy
EXCEPT:
a) cardiac output increases 33-45%
b) stroke volume increases 10-30%
c) heart rate decreases 12-18 beats per minute
d) systolic blood pressure decreases 4-5 mmHg
e) diastolic blood pressure decreases 8-15 mmHg
21) A couple are unable to conceive after 1 year. Of the following
statistics regarding infertility, which are correct:
a) 10-15% of couples are infertile
b) only 40% of couples achieve pregnancy within 6 months of
trying
c) only 20% achieve pregnancy within 1 year of trying
d) male factors are responsible for over half of infertility causes
e) infertility is labelled after failure to conceive within 2 years of
trying
22) Vaginal discharge which is fishy in odor and associated with
>20% clue cells on microscopy will not be associated with which of
the following:
a) be due to gardnerella vaginalis overgrowth
b) can be treated with metronidazole or clindamycin
c) with the addition of KOH may see hyphae or spores
d) must be treated in all pregnant women, including
asymptomatic
e) is rarely associated with inflamed or itchy vulva
23) In the treatment of an ectopic pregnancy with methotrexate all of
the following are true EXCEPT:
a) the pregnancy must not have ruptured
b) it must be less than 5cm in size
c) patient should be clinically stable
d) compliance and f/u are ensured
e) no fetal heart activity has been noted
24) Which of the following is associated with breech presentations?
a) fetal heart sounds heard best in the upper abdomen
b) brachial plexus injury
c) prolapse of the umbilical cord
d) increased fetal mortality
e) all of the above

28) On colposcopy, the transformation zone:


a) contains columnar epithelium
b) is the area between original squamous epithelium and columnar
epithelium
c) contains metaplastic epithelium
d) contains atypical blood vessels
29) Complications of cesarean section include each of the
following EXCEPT:
a) hemorrhage
b) infection
c) laceration of the fetus
d) subsequent rupture of the uterine scar
e) inversion of the uterus
30) Regarding laparoscopy:
a) it is contraindicated in patients who are menstruating.
b) patients with intestinal obstruction are still able to undergo
laparoscopy
c) the most common indication for therapeutic laparoscopy is tubal
sterilization
d) all of the above are incorrect
31) Causes of secondary amenorrhea include:
a) Turner syndrome
b) anorexia
c) androgen insensitivity syndrome
d) gonadal dysgenesis
e) imperforate hymen
32) Risk factors for cervical carcinoma includes all of the
following EXCEPT:
a) human papilloma virus type 16 and 18
b) early age at first intercourse
c) smoking
d) nulliparity
e) multiple sexual partners
33) A 35 year-old G0P0 has severe pain during menses which radiates
into the anal region. On bimanual exam, she has a small, retroverted
uterus and tender nodules can be felt on palpation of the uterosacral
ligaments. Which of the following conditions does she most likely
have:
a) chronic PID
b) adenomyosis
c) fibroids
d) endometriosis
e) uterine carcinoma
34) Which of the following statements is correct?
a) cervical carcinoma is the most common gynecological
malignancy in Canada
b) lichen sclerosis is treated with estrogen cream
c) colposcopy is required after a finding of mild cervical
dysplasia on a routine Pap smear
d) follicular cysts typically regress with the following cycle
e) leiomyomata are malignant tumours

Toronto Notes 2005

Obstetrics and Gynecology

Sample Questions 13

35) If a mother nurses her child, the time of reappearance of her


menses is uncertain but usually occurs after her delivery by:
a) 1-2 months
b) 2-3 months
c) 3-4 months
d) 4-5 months
e) > 5 months

43) The appropriate next step in a woman found to have 1 abnormal


Pap smear showing mild dysplasia (CIN1) is:
a) repeat in 1 year's time
b) repeat in 4-6 months time with colposcopy if abnormal
c) send patient directly for colposcopy and LEEP excision
d) send for colposcopy after 1 abnormal Pap
e) repeat Pap immediately and treat for HPV infection

36) Which of the following statements regarding malignant cervical


lesions is true?
a) 95% are squamous cell carcinoma
b) CA-125 levels to monitor treatment effectiveness are indicated
c) the majority of lesions arise outside the transformation zone of
the cervix
d) they are not associated with HPV infection
e) treatment for stage 4 disease radical hysterectomy with
chemotherapy

44) The commonest cause of disseminated intravascular


coagulation during pregnancy is:
a) amniotic fluid embolism
b) abruptio placentae
c) placenta previa
d) missed abortion
e) chorioamnionitis

37) In endometriosis, the most common location for disease is:


a) broad ligament
b) cul-de-sac
c) ovaries
d) appendix
e) uterosacral ligament
38) All of the following increase the risk of developing ovarian cancer
EXCEPT:
a) family history
b) BCP
c) nulliparity
d) late menopause
e) caucasian

45) Contraindications to vaginal delivery include each of the


following EXCEPT:
a) previous classical cesarean section
b) squamous cell carcinoma in situ of the cervix
c) total placenta previa
d) transverse lie with ruptured membranes
e) primigravida breech with cephalo-pelvic disproportion
46) In a differential diagnosis of third trimester bleed, which of the
following should not be included:
a) placenta previa
b) vasa previa
c) abruptio placentae
d) threatened abortion
e) marginal sinus hemorrhage

39) In the first 100 days of pregnancy, human chorionic gonadotropin


titre doubles:
a) every day
b) every week
c) every 2 days
d) every month
e) every 2 weeks

47) A 41 year-old woman presents with obesity, hirsutism and


oligomenorrhea, which of the following tests will give the least
useful information?
a) FSH
b) LH
c) testosterone
d) TSH
e) urine free cortisol

40) A young female enters your office wanting to start oral


contraceptive pills. You remind yourself of the reasons she may
not start the pill which are:
a) impaired liver function
b) undiagnosed abnormal uterine bleeding
c) congenital hyperlipidemia
d) past history thrombophlebitis
e) all of the above.

48) With respect to androgen insensitivity syndrome, which of the


following is true?
a) genotype is XY
b) breast development and uterus are absent
c) serum testosterone is below normal male range
d) secondary sex characteristics are male
e) the condition is autosomal recessive

41) Which of the following statements regarding endometriosis is


incorrect?
a) theories proposed to explain the histogenesis of endometriosis
include retrograde menstruation and coelemic metaplasia
b) CA-125 is often useful in making the diagnosis of
endometriosis
c) symptoms of endometriosis may include urinary frequency
and diarrhea
d) the differential diagnosis of endometriomas include
hemorrhagic
corpus luteum cysts as well as neoplasms
e) the risk of endometriosis is several fold greater if there is a
first degree relative with this condition
42) A 55 year-old female presents with a "period-like" vaginal
bleeding for 5 days that stopped spontaneously. Your approach
to the problem would be to:
a) reassure the patient and manage expectantly
b) administer provera 10 mg od for 15 days
c) administer cyclic estrogen plus progesterone
d) obtain endometrial tissue
e) perform a pap smear plus endocervical curretage

49) The new selective estrogen receptor modulators have all of the
following benefits EXCEPT:
a) no agonistic effect on breasts
b) provides estrogenic benefit on bone
c) mimics estrogenic effect on cardiovascular status
d) no agonistic effect on uterine tissue
e) provides relief of hot flushes associated with menopause
50) Regarding birth control, which of the following is false?
a) it is contraindicated in women over age 35 who are smokers
b) its efficacy may be decreased when taking antibiotics
c) reduces the risk of ovarian carcinoma
d) is associated with only a 10% failure rate
e) can be associated with breast changes and weight gain.

14 Sample Questions

Pediatrics

Toronto Notes 2005

Pediatrics
1) A 6 month-old infant presents in the winter with fever, cough,
wheezing, tachypnea and decreased appetite. A chest
radiograph shows hyperaeration and streaky perihilar infiltrates
bilaterally. You diagnose bronchopneumonia. Which organism
would most likely be causing this child's infection?
a) Chlamydia pneumoniae
b) Mycoplasma pneumoniae
c) Streptococcus pneumoniae
d) Haemophilus influenzae
e) Respiratory syncytial virus
The following case pertains to questions 2 and 3:
A 12 month-old girl is brought to the emergency department for
the second time in 2 days for vomiting and passage of 8 to 10
watery stools per day.
2) Of the following, which provides the best estimate of the
patients volume deficit:
a) weight change since the beginning of the illness
b) hydration of mucous membranes, skin turgor, and
level of consciousness
c) pulse, blood pressure, and peripheral capillary filling time
d) serum electrolytes
e) serum urea nitrogen and creatinine levels
3) The patient has lost 0.6 kg. She is moderately lethargic and has
dry mucous membranes and reduced skin turgor. Blood pressure
is 80/40 mm Hg, and pulse is 120 per minute; capillary refill is
reasonably brisk. Lab studies reveal: sodium 131, potassium 4.8,
chloride 101, bicarbonate 16 mEq/L, urea nitrogen 24 mg/dL and
creatinine 0.6 mg/dL. The best strategy for managing this child is
to:
a) hospitalize for administration of IV fluid therapy
b) administer an oral rehydrating solution while the
child is under medical supervision for 4-6 hours
c) instruct the parents about oral rehydration at home
d) hospitalize after giving 20 mL/kg of 0.9% saline IV
e) instruct parents on use of soy formula
4) Which of the following statements about Attention Deficit and
Hyperactivity Disorders (ADHD) is INCORRECT?
a) ADHD is more commonly seen in boys than girls
b) deafness and seizure disorders must be excluded before
the diagnosis of ADHD is made
c) three characteristic features of ADHD are inattention,
hyperactivity, and mental retardation
d) ritalin is effective in the treatment of about 70% of cases of
ADHD
e) side effects of Ritalin include poor growth, insomnia, and tics
5) A 6 year-old child is brought into the Emergency with his father
because of a fractured arm. Although his father states that the
child fell off his bike, the fracture does not seem consistent with
this story. On inspection, you discover that the child seems to
have a number of old bruises on his back and on his legs, some
of which seem reminiscent of finger markings. Which of the
following steps is not appropriate in this circumstance?
a) let the child go home with his father since you cannot really
prove anything
b) document the injuries that you saw with possible
photographs
c) call the CAS and inform them
d) try to interview the child alone
e) do additional investigations (e.g. skeletal survey)
6) A 14 year-old girl has refused to go to school 3 times in the past
4 months. She says, "My tummy hurts," but she cannot point to
where it bothers her. Her appetite is good, her bowel movements
are normal, and she is sleeping well at night. She is a
healthy-looking girl with no abnormal findings on physical exam.
What is the most likely diagnosis?
a) a benign abdominal mass compressing her duodenum
b) diaphragmatic hernia
c) somatoform disorder
d) gastroesophageal reflux disease
e) early peptic ulcer disease

7) Which of the following investigations is most helpful in the


assessment of a child presenting with an acute asthma attack?
a) white cell count and differential
b) arterial blood gases
c) chest x-ray
d) pulmonary function tests
e) O2 sat. monitoring
8) A 6 year-old boy is brought to emergency with a 5 day history of
fever, cough, and poor appetite, but no vomiting. On exam, he
appears unwell, is febrile, and has crusty nasal discharge and a
wet-sounding cough. Chest x-ray reveals a pulmonary infiltrate in
the right middle lobe. He is admitted to hospital for pneumonia.
What is the best management for this child?
a) observe for 24 hours with IV fluids only
b) observe for 24 hours with IV fluids and acetaminophen
c) give oral amoxicillin and acetaminophen
d) give IV ampicillin and oral acetaminophen
e) give oral erythromycin and acetaminophen
9) Side effects of methylphenidate (Ritalin) therapy for hyperactivity
include each of the following EXCEPT:
a) anorexia
b) insomnia
c) tachycardia
d) dependence (addiction)
e) growth suppression
10) In comparing breast milk and formula, which of the following
statements is NOT correct:
a) breast milk has a higher percentage of protein
b) breast milk has whey:casein ratio of 60:40
c) breast milk contains leukocytes, complement and lysozymes
d) breast milk has a lower concentration of iron than iron
fortified formulae
e) breast milk has an optimal calcium:phosphorus ratio of 2:1
11) Most umbilical hernias in children:
a) need strapping
b) resolve spontaneously
c) require elective surgery
d) require immediate surgery
e) are associated with a higher incidence of inguinal hernias
12) A newborn male spits up his first feeding and develops bilious
emesis with subsequent feedings. On physical exam he appears
ill, has a scaphoid abdomen and absent bowel sounds.
Abdominal x-ray shows air in the proximal small bowel, but a
paucity of air in the distal digestive tract. The most likely cause
for this infant's vomiting and clinical finding is:
a) antral web
b) choldochal cyst
c) Hirschsprung disease
d) tracheoesophageal fistula
e) volvulus
13) Breastfeeding is contraindicated if the mother has:
a) acute EBV infection
b) acute HepA infection
c) chronic HepB infection
d) asymptomatic HIV infection
e) none of the above
14) A 10 month-old child develops low-grade fever and sunburn-like
erythema over much of the body, but most prominently
in the intertriginous areas. Within 36 hours, sheet-like
desquamation is noted in the flexures and around the mouth.
The mucous membranes are spared. Which of the following is
the most likely diagnosis:
a) Kawasaki disease
b) staphylococcal scalded skin syndrome
c) Stevens-Johnson syndrome
d) toxic epidermal necrolysis
e) toxic shock syndrome

Toronto Notes 2005

Pediatrics

15) By one year of age, a child is able to have all of the following
foods except:
a) breast milk
b) peeled fruits
c) cheese
d) peanuts
e) eggs
16) Central cyanosis in the newborn infant is most often caused by:
a) congenital heart disease
b) lung disease
c) central nervous system disease
d) methemoglobinemia
e) hypoglycemia
17) In a premature infant who is suspected of having necrotizing
enterocolitis (NEC), each of the following is correct EXCEPT:
a) Septicemia is associated with an increased risk of NEC
b) NEC is thought to be caused by systemic hypertension
c) the finding of air in the portal vein indicates severe illness
d) respiratory distress increases the risk of NEC
e) apgar scores inversely correlate with the risk of NEC
18) Which of the following is the most significant risk factor for the
development of childhood asthma:
a) family social background
b) parental asthma
c) stress in the family
d) parental smoking
e) presence of pets in the house
19) Which of the following statements about sickle cell disease is true?
a) all patients with sickle cell disease have a homozygous HbSS
genotype
b) sickle cell disease causes a severe chronic anemia that is not r
outinely transfusion dependent
c) patients have increased susceptibility to infection by
nonencapsulated organisms
d) patients usually present with sickle cell crises within one month
of age
e) splenic dysfunction usually does not occur until the child enters
his/her teens

Sample Questions 15

23) When prophylactic antibiotic therapy is used for tooth


extraction in a patient with a ventricular septal defect:
a) therapy is started 24 hours prior to the procedure
b) a throat swab should be taken prior to the procedure
c) a second generation cephalosporin (cefuroxime) is the
therapy of choice
d) therapy is given for one week following the extraction
e) erythromycin is the drug of choice for those allergic to
penicillin
24) Which of the following statements about stuttering in a
4 year-old is incorrect:
a) it is characterized by intermittent difficulty in producing a
smooth flow of speech
b) it is more than 3 times more frequent in girls than in boys
c) it is exacerbated by anxiety
d) more than 30% of children who stutter recover
spontaneously
e) none of the above
25) A woman who is positive for hepatitis B surface antigen
(HBsAg), but negative for hepatitis B antigen (HBeAg),
delivers at term. What would be the best management for
this woman's infant?
a) administer gamma globulin intramuscularly immediately
and at 1 month of age
b) administer hepatitis B (HB) vaccine immediately and at 1
month and 6 months of age
c) administer hepatitis B immune globulin (HBIG) if cord
blood is positive for HbsAg
d) administer HBIG and HB vaccine immediately, and HB
vaccine again at 1 month and 6 months of age
e) advise mother that breastfeeding is contraindicated
26) Regarding sexual abuse of children, each of the following
statements is true EXCEPT:
a) no genital injury is found in the majority of patients
b) father-daughter incest is more common than brother-sister
c) most assailants are unknown to the victim
d) laboratory findings usually show no presence of sperm
e) half of the abused children come from single parent
families

20) Which of the following is false about physiological jaundice?


a) it rarely presents before the age of 24 hours
b) it may cause kernicterus
c) in premature infants it may persist for 3-4 weeks
d) it is mainly due to temporarily impaired hepatic clearance of
bilirubin
e) it does not always require treatment with phototherapy

27) The viral infection most likely to cause CNS involvement and
focal neurological findings is :
a) coxsackievirus
b) herpes simplex
c) enterovirus
d) rabies
e) rhinovirus

21) Which of the following statements is true?


a) cancer is the second most common cause of death in children
b) Hodgkin's lymphoma is the most common childhood cancer
c) hyperploidy in leukemic cells is a poor prognostic indicator
d) Wilm's tumor is rarely associated with other congenital
abnormalities
e) neuroblastomas usually occur in late adolescence

28) Which of the following predisposes infants to chronic otitis media?


a) bottle-feeding in upright position
b) abnormal tympanic membrane formation
c) environmental factors such as daycare and passive smoking
d) allergies
e) none of the above

22) A 5 year-old girl with hypogammaglobulinemia and absent


immunoglobulin A (IgA) receives infusions of immune
globulin monthly. Shortly after her most recent infusion
began, she developed hypotension, wheezing, and several
urticarial lesions. Which of the following best explains her
reaction?
a) anaphylactic reaction due to IgE anti-IgA antibodies
b) Gram-negative sepsis due to contaminated immune globulin
c) idiosyncratic reaction due to rapid infusion of immune
globulin
d) serum sickness reaction from foreign serum in the immune
globulin
e) transfusion reaction due to ABO incompatibility

29) A 13 month-old infant boy has chronic diarrhea, poor appetite,


irritability and growth failure. He had been well previously,
developing normally until diarrhea began at 3 months of age.
Findings include weight loss less than 5th percentile and length
at 25th percentile, cachectic appearance, wasted extremities and
protuberant abdomen. His labs are: albumin 23, protein 40.
Stool positive for reducing sugars and negative for enteric
pathogens and ova and parasites. These findings are most
consistent with:
a) celiac disease
b) cow milk allergy
c) Crohn disease
d) Cystic Fibrosis
e) none of the above

16 Sample Questions

Pediatrics

Toronto Notes 2005

30) All infants less than 3 months of age who have fever and no
localizing signs should have all of the following evaluations
EXCEPT:
a) blood cultures
b) stool cultures
c) chest x-ray
d) CBC (with differential)
e) urine culture

38) An 8 year-old boy has had paroxysmal abdominal pain since his
parents separated 6 months ago. Which of the following
symptoms would support and organic basis for his disease?
a) headaches accompanies the pain
b) pain located in the periumbilical region
c) pain awakens child at night
d) symptoms last < 1hr
e) none of the above

31) All the following statements regarding transient tachypnea of the


newborn (TTN) is true EXCEPT:
a) infants born by C-section are at increased risk for developing
TTN
b) residual pulmonary function disability is common among
infants who have TTN
c) the incidence of TTN is higher than Respiratory Distress
Syndrome (RDS) among term infants
d) TTN shows marked improvement with 12-24 hrs
e) chest x-ray findings consisntent with TTN include increased
pulmonary vasculature and fluid in the fissures

39) The most common cause of chronic diarrhea in a


6 month - 36 month old child is:
a) toddlers diarrhea
b) disaccharidase deficiency
c) enteric infection
d) malabsorption
e) protein intolerance

32) Common signs of heart failure in children include all of the


following EXCEPT:
a) decreased exercise tolerance
b) fatigue
c) failure to thrive
d) frequent URTIs
e) peripheral edema
33) Antibiotic prophylaxis against infective endocarditis is required for
all of the following EXCEPT:
a) rheumatic valve lesions
b) prosthetic heart valves
c) isolated secundum ASD
d) pacemaker leads
e) PDA
34) Causes of microcytic anemia include all of the following EXCEPT:
a) excessive cow's milk intake
b) iron deficiency
c) folic acid deficiency
d) thalassemias
e) lead poisoning
35) The latest time after the onset of GABHS pharyngitis that initiation
of penicillin therapy can be expected to prevent acute rheumatic
fever is:
a) 24 hrs
b) 48 hrs
c) 96 hrs
d) 9 days
e) 2 weeks
36) Fetal tricuspid valve abnormalities are associated with
maternal use of which of the following:
a) lithium
b) warfarin
c) chloramphenicol
d) estrogen
e) heroin
37) Which of the following is most likely to provide the basis for
making the correct diagnosis in an infant or child who has failure to
thrive (FTT) ?
a) blood chemistries
b) cultures
c) history and physical
d) radiograph studies
e) none of the above

40) Which of the following is NOT required for a diagnosis of


juvenile rheumatoid arthritis?
a) arthritis in at least one joint
b) arthritis lasting for at least 6 weeks
c) positive rheumatoid factor
d) onset before the age of 16
e) other causes of arthritis excluded
41) A child presents with bilateral shin pain. Which of the following
suggests that this is NOT growing pains?
a) pain is poorly localized
b) pain awakens the child at night
c) no fever or rash
d) pain abates with reassurance and massage
e) child may limp in the morning from stiffness
42) Which of the following statements about precocious puberty is
true?
a) the cause is never found in most cases
b) the children are always shorter than average
c) the incidence is the same for males and females
d) ketoconazole is the treatment of choice for females
e) all girls with breast development before age 10 require
karyotyping for Turner syndrome
43) All of the following groups are contraindicated to receive the oral
polio vaccine EXCEPT:
a) pregnant women
b) asymptomatic HIV-positive children
c) household contacts of immunodeficient children
d) unimmunized adults at future risk of being exposed to the
polio virus
e) children with agammaglobulinemia
44) Which of the following steps is NOT indicated in the
management of croup?
a) keeping the child calm
b) hydration
c) antipyretics
d) antibiotics
e) humidified oxygen
45) Which of the following pairs shows the CORRECT stage of
normal development?
a) 6 months - pulls to stand
b) 12 months - pincer grasp
c) 18 months - handedness
d) 2 years - draws a cross
e) 3 years - tells a story
46) Which of the following is NOT characteristic of a functional
murmur?
a) pansystolic murmur
b) murmur varies with position
c) variably split S2
d) murmur becomes louder with fever
e) no extra clicks

Toronto Notes 2005

Pediatrics

47) Which of the following is the most common congenital heart


lesion in children?
a) ASD
b) VSD
c) PDA
d) Tetralogy of Fallot
e) coarctation of the aorta
48) Choose the INCORRECT statement about breastfeeding:
a) for healthy term babies, breastfeeding is recommended over
formula feeding
b) breastmilk contains more amino acids than cows milk
c) exclusively breastfed babies should be supplemented with
iron after 6 months of age
d) assessment of adequate intake can be done by counting
the number of wet diapers in a day
e) an HIV-positive mother should not breastfeed
49) All of the following are characteristics of a child with Pervasive
Developmental Disorder (PDD) EXCEPT:
a) abnormal speech pattern
b) stereotypic behaviours, such as head-banging or hand-flapping
c) abnormal cognitive function
d) consuming interest in one topic or activity
e) tendency to reciprocate in peer interactions
50) All of the following are appropriate in the initial assessment of a
child with failure to thrive EXCEPT:
a) diet history
b) social history
c) measurement of height, weight and head circumference
d) growth hormone levels
e) bone age x-ray
51) Which of the following statements about a child with short stature is
CORRECT?
a) the bone age is delayed in a child with constitutional growth
delay
b) height crosses a major percentile line on the childs growth chart
c) weight is more affected than height in a child with an endocrine
deficiency
d) karyotyping is part of the routine investigation of all children
with short stature
e) growth hormone replacement is helpful in a child with familial
growth delay
52) An 8 year-old boy who has been continent of urine between the
ages of 4 and 5 years comes in because of recent onset of
enuresis. Which of the following statements is false?
a) it may be precipitated by a family crisis
b) it does not require a specific treatment
c) it can lead to poor self esteem
e) it responds best to individual psychotherapy
e) it can be secondary to diseases such as diabetes mellitus
53) Which of the following is NOT correct about the pertussis
vaccine:
a) delaying the primary series until the age of 1 year will not
reduce the risk of a seizure
b) the commonly quoted risk of 1:330,000 of brain damage has
no basis in fact
c) it can cause febrile seizures
d) it may cause infantile spasms or sudden infant death
syndrome
e) the risk of other forms of neurologic illness eg. transverse
myelitis is extremely small
54) The most common cause of apnea in infants less than
6 months is:
a) encephalitis
b) seizure disorder
c) cardiac arrhythmia
d) milk allergy
e) gastroesophageal reflux

Sample Questions 17

55) A 7 day-old breastfed infant born at term has had decreased


appetite, irritability, and vomiting for 24 hours. On physical
examination, the infant appears listless. Respiratory rate is
40/min; heart rate, 160/min; and blood pressure, 68/38 mm Hg.
he skin and sclerae are icteric but no other abnormalities are
noted. Laboratory studies reveal: hemoglobin 120 g/L; total
bilir bin, 270 umol/L; and direct bilirubin, 135 umol/L.
Urinalysis is negative for reducing substances. Which of the
following is the most likely diagnosis:
a) bacterial sepsis
b) blood group incompatibility
c) breast milk jaundice
d) hypothyroidism
e) intrauterine infection
56) Each of the following statements about rotavirus gastroenteritis
in children is correct except:
a) it usually occurs in the winter months
b) it is self-limited
c) it is often associated with respiratory symptoms
d) it is often accompanied by fever and vomiting
e) it is often associated with grossly bloody diarrhea
57) The most common cause of deterioration of diabetic control
in an 8 year-old boy is:
a) change in diet
b) lack of compliance
c) emotional stress
d) acute infection
e) change in exercise level
58) A child born at 32-weeks gestation is brought to your office at
2 months of age for a routine check-up. The child has an
upper respiratory tract infection but is afebrile. What decision
would you make regarding the child's first vaccination?
a) wait until the child is two months older to administer the
first vaccination
b) wait until the child is clinically well and then vaccinate
c) administer the first vaccination
d) consult a pediatrician
e) none of the above
59) The most common cause of hemoptysis in a 6 year-old boy is:
a) cystic fibrosis
b) pneumonia
c) foreign body
d) tracheobronchitis
e) neoplasm
60) A 15 month-old child has spastic cerebral palsy. Which of the
following is most suggestive of perinatal asphyxia as the cause
for this condition:
a) evidence of periventricular calcificatons at birth
b) microcephaly at birth
c) recurrent seizures in the newborn period
d) retinopathy of prematurity
e) rigidity and spasticity
61) Which of the following findings is least consistent with a
diagnosis of congenital deafness in a 16 month-old infant:
a) smiling responsively at 1 month
b) cooing at 2 months
c) babbling at 3 to 6 months
d) imitating speech sounds at 9 months
e) no clearly enunciated words at 15 months
62) Which of the following genetic diseases is incorrectly
linked with its transmission pattern:
a) Duchenne muscular dystrophy = sex-linked recessive
b) pyloric stenosis = polygenetic inheritance
c) cerebral palsy = polygenetic inheritance
d) sickle cell disease = autosomal codominant
e) color blindness = autosomal recessive

18 Sample Questions

Pediatrics

Toronto Notes 2005

63) Each of the following is an indication for tonsillectomy except:


a) recurrent episodes of otitis media
b) peritonsillar abscess
c) carcinoma of the tonsil
d) hypertrophy with airway obstruction
e) recurrent (at least six) episodes of group A Strep
tonsillitis within one year

70) During the newborn period, physical characteristics suggestive


of prematurity include each of the following except:
a) a prominent diastasis of the rectus muscles
b) scant ear cartilage
c) absent breast buds
d) thin skin
e) hypotonic posture at rest

64) A healthy, full-term, breastfed infant is jaundiced at 24 hours of


age. The indirect serum bilirubin level is 210 umol/L. The
mother is blood type O, Rh positive. Which of the following is
the most appropriate step:
a) culture the blood
b) determine glucose-6-phosphate dehydrogenase activity
c) interrupt breast feeding
d) obtain a reticulocyte count
e) obtain blood type and Coombs test

71) A 5 year-old child is diagnosed as having juvenile rheumatoid


arthritis. Which of the following is the least likely side effect of
therapy with a non-steroidal anti-inflammatory drug:
a) abdominal pain
b) lymphadenopathy
c) tinnitus
d) anemia
e) rash

65) A 6-week-old male infant develops recurrent episodes of


vomiting with dehydration. Each of the following would be
consistent with a diagnosis of congenital hypertrophic pyloric
stenosis except:
a) non-bile stained vomitus after feeding
b) a metabolic acidosis and hyperkalemia
c) a history of increasing constipation
d) good appetite
e) visible epigastric peristaltic waves
66) Each of the following statements regarding AIDS in infants and
children is correct except:
a) the risk of nosocomial transmission is low
b) function of both T and B lymphocytes is impaired
c) hepatosplenomegaly is a common associated finding
d) poor growth occurs frequently
e) chronic pneumonitis is a rare complication
67) A 14 year-old boy is concerned about his height. Physical
examination reveals Tanner stage II pubic hair and genital
development. Height and weight are at the 10th percentile for
age. Which of the following is the most accurate statement
about his growth:
a) he is taller than the average 14 year-old girl
b) he will be taller than his peers by age 16
c) he has not reached his peak height velocity
d) he will stop growing by age 17
e) his adult height will be at the 10th percentile
68) A 3 year-old girl has a urinary tract infection and receives
appropriate antibiotic therapy. Following treatment,
ultrasonography and voiding cystourethrography reveal normal
findings. Which of the following would be the best
management of this patient's urinary tract problem?
a) administrate polyvalent vaccine against multiple enteric
organisms
b) begin daily prophylaxis now and treat each future episode
of infection
c) obtain intravenous pyelography
d) treat each future episode of infection, but withhold
prophylaxis
e) none of the above
69) A 10 year-old boy with cough, fever and night sweats has a
positive skin test for tuberculosis. A chest x-ray reveals minimal
hilar adenopathy and a small pulmonary infiltrate. The current
recommended treatment is:
a) isoniazid alone
b) isoniazid plus rifampin and pyrazinamide
c) isoniazid plus rifampin and streptomycin
d) rifampin and corticosteroids
e) rifampin and ethambutol

72) Which of the following statements about tumours in childhood


is false:
a) ALL is the most common malignancy
b) brain tumours are the most common solid tumours
c) malignancy is the third most common cause of death after
accidents and homicides
d) neuroblastoma spontaneously regresses
e) blacks and whites do not have equal incidence of tumours
73) Which of the following is a characteristic chest x-ray
appearance of respiratory distress syndrome (RDS):
a) air bronchograms
b) fluid in fissure
c) patchy infiltrate
d) atelectasis
e) enlarged heart
74) An 8 year-old boy presents with sudden onset of pain in the
distal femur. His parents recall that he was hit with a hockey
stick during a street hockey game last week. The most likely
diagnosis is:
a) osteomyelitis
b) fracture of the distal femur
c) juvenile rheumatoid arthritis
d) septic arthritis
e) Blounts disease
75) The most likely diagnosis in a 3 month-old with a left flank
mass is:
a) renal cell carcinoma
b) Wilms tumor
c) neuroblastoma
d) renal stones
e) polycystic kidney disease
76) Which of the following congenital anomalies usually resolves
spontaneously:
a) club foot
b) genu varus
c) tibial torsion
d) flexible flat feet
e) genu valgus
77) Which of the following is the most important factor in
determining whether a child will die of measles:
a) gender
b) age
c) nutritional status
d) muscle mass
e) fitness level

Toronto Notes 2005

78) At 36 hours of age a full-term infant has not yet passed


meconium. Physical examination reveals jaundice and
abdominal distention. Radiographs of the abdomen show
dilated loops of bowel and calcifications in the scrotum. A
meconium plug is passed after rectal examination. Which of
the following would be the most definitive diagnostic test to
order:
a) barium enema
b) sweat chloride test
c) ultrasound of the abdomen
d) urine calcium excretion
e) voiding cystourethrogram
79) Organic causes in the differential diagnosis of chronic
abdominal pain includes all of the following except:
a) UTI
b) lactose intolerance
c) chronic giardiasis
d) chronic appendicitis
e) inflammatory bowel disease
80) Which of the following problems is most likely to occur in a
12 month-old infant who began taking whole cow milk at
6 months of age:
a) increased frequency of otitis media
b) wheezing episodes
c) nasal congestion
d) decreased serum ferritin level
e) diaper dermatitis

Pediatrics

Sample Questions 19

81) A 1 year-old child with a rapid pulse, slightly low blood


pressure, cool extremities, oliguria and dry oral mucosa should
be considered:
a) 5% dehydrated
b) 10% dehydrated
c) 15% dehydrated
d) 20% dehydrated
82) All of the following are indications for tympanostomy tubes
adenoidectomy in a child except:
a) persistent 3 month effusion
b) failure of prophylactic medications
c) associated hearing loss
d) recurrent acute otitis media
83) A 2 year-old with fever is seen in hospital following a
generalized seizure. He also has foul smelling, blood-tinged
diarrhea. The culture of the stool is most likely to grow:
a) Campylobacter species
b) E. coli
c) Entamoeba histolytica
d) Salmonella species
e) Shigella species

20 Sample Questions

Psychiatry

Toronto Notes 2005

Psychiatry
1) Monoamine oxidase inhibitor drugs are used in the treatment of
depression because they increase synaptic levels of:
a) gamma-aminobutyric acid (GABA)
b) histamine
c) acetylcholine
d) norepinephrine
e) somatostatin
2) Neuropsychological effects of hallucinogens may include all of the
following EXCEPT:
a) miosis
b) tremor
c) hyper-reflexia
d) uncoordination
e) blurred vision
3) Cocaine withdrawal can include all of the following EXCEPT:
a) Crash sleep
b) anergia
c) anhedonia
d) euphoria
e) continued craving
4) Alcohol withdrawal includes all of the following EXCEPT:
a) autonomic hyperactivity
b) tremor
c) starts within 2-4 hours after prolonged drinking
d) nausea
e) irritability
5) Which would not be considered a risk factor for suicide in patients
presenting with suicidal ideation:
a) substance abuse
b) male gender
c) lack of social supports
d) unsuccessful attempt at suicide in the past
e) childless marriage
6) A 54 year-old man has become forgetful, preoccupied,
withdrawn, irritable and dishevelled. His physical examination
was normal. The patient had been with his company for
twenty-two years and was considered an excellent employee.
Which of the following is the most likely diagnosis:
a) multi-infarct dementia
b) hypothyroidism
c) schizophrenia
d) alcoholism
e) major depression
7) Which of the following is correct about depression in
children:
a) family therapy should be avoided because it scapegoats
a child who is already vulnerable
b) symptoms may manifest as antisocial behaviour
c) antidepressants generally are not effective in children
d) the suicide rate in children aged 8-13 is higher than
it is in older adolescents
e) depression in children has been shown to be a prodrome
to the later development of schizophrenia
8) All of the following are classified as paraphilias EXCEPT:
a) fetishism
b) homosexuality
c) exhibitionism
d) sexual sadism
e) transvestism

9) A 32 year-old engineer has been uncharacteristically active for


several weeks. He spends most of his time at work and gets
little sleep. He has told another engineer that he is involved in
a research project that will earn me the Nobel Prize.
Expensive research equipment keeps arriving at his office. The
engineer is irritable, and it is hard to hold his attention. A
classmate from graduate school recalls that the patient
behaved in a similar manner twice during stressful periods at
school.
Longterm drug therapy for this patient would likely include:
a) haloperidol
b) valproic acid
c) clozapine
d) ascorbic acid
e) chlordiazepoxide
10) Elderly male depressives typically present with all of the
following EXCEPT:
a) importuning
b) anxiety
c) weight loss
d) little suicide risk
e) insomnia
11) From among the drugs listed below, which would be the cause
for most concern in an overdose:
a) paroxetine (SSRI)
b) amitriptyline (tricyclic)
c) diazepam (benzodiazepine)
d) chlorpromazine (phenothiazine)
e) fluoxetine (SSRI)
12) Which of the following statements about schizophrenia is false?
a) male schizophrenics experience their first psychotic
episode at a younger age than women
b) male schizophrenics are more frequently hospitalized than
female schizophrenics
c) compared to young female schizophrenics, young male
schizophrenics are at increased risk of movement disorders
secondary to neuroleptics
d) in women, the symptoms of schizophrenia tend to worsen
after menopause
e) all of the above statements are true
13) A 29 year-old school teacher who lives alone is brought to the
emergency room because she has become increasingly suspicious,
hyperactive, and anorexic over the past two days. She believes
that people in the neighbourhood are out to get me. She has
not slept in 2 nights. She reports seeing snakes crawling on the
wall. Based on this information, the most likely diagnosis of the
womans problem is:
a) anorexia nervosa
b) cocaine withdrawal
c) paranoid personality
d) psychostimulant abuse
e) shared paranoid disorder
14) Anti-alpha-1-adrenergic blockade causes:
a) nausea
b) constipation
c) orthostatic hypotension
d) dry mouth
e) drowsiness
15) The following are common side effects of SSRIs EXCEPT:
a) Headache
b) Sexual dysfunction
c) Vomiting
d) Anorexia
e) Orthostatic hypotension

Toronto Notes 2005

Psychiatry

Sample Questions 21

16) Clozapine is the neuroleptic of choice for schizophrenia when:


a) the patient shows no evidence of tardive dyskinesia
b) the patient has not improved with adequate
dosages for 6 weeks of 3 other antipsychotics
c) the patient is under fifteen years of age
d) the patient has a WBC count of >10 000
e) the patient is very sensitive to anticholinergic side effects

23) Hypnosis has been used successfully in all of the following


conditions EXCEPT:
a) pain
b) phobia
c) paranoia
d) anxiety
e) smoking

17) Frequent conditions appearing comorbidly with ADHD include:


1) oppositional defiant and conduct disorders
2) anxiety disorder
3) learning disabilities
4) tic disorders
5) language disorders

24) Cognitive therapy helps to correct which of the following


cognitive distortions:
a) depersonalization
b) psychotic thinking
c) over-generalizations
d) selective inference
e) hallucinations

Which are correct:


a) only 1 is correct
b) 1, 2, and 3 are correct
c) 2 and 4 are correct
d) all are true
18) A 35 year-old man presents to the emergency room with
suicidal ideation. He describes significant stress due to
recent job loss and financial hardship. Further inquiry reveals
a history of repeated job loss, fraud charges, and frequent
arm slashing to decrease stress. The man was not disruptive
as a child. The most likely diagnosis is:
a) borderline personality disorder
b) antisocial personality disorder
c) adjustment disorder
d) dysthymic disorder
e) schizophrenia
19) Which of the following statements concerning anorexia
nervosa and bulemia nervosa is false:
a) patients with either of these eating disorders are
preoccupied with weight, food, and body shape
b) bulemia nervosa is more prevalent than is anorexia nervosa
c) both of these eating disorders are more common in
females than in males
d) bulemia nervosa often presents earlier in adolescence than
does anorexia nervosa
e) bulemic symptoms may occur in both bulimia nervosa and
anorexia nervosa
20) A pattern of unstable but intense interpersonal relationships,
impulsivity, inappropriately intense anger, identity disturbance,
affective instability, and problems with being alone suggest a
diagnosis of:
a) antisocial personality disorder
b) narcissistic personality disorder
c) histrionic personality disorder
d) schizoid personality disorder
e) borderline personality disorder
21) The criteria for diagnosis of a factitious disorder include:
a) intentional production or feigning of physical signs or
symptoms
b) absence of secondary gain
c) possibility of economic gain
d) (a) and (b)
e) (a) and (c)
22) A patient with a fear of heights is brought to the top of a tall
building and required to remain there as long as necessary
for the anxiety to dissipate. This is an example of:
a) graded exposure
b) participant modelling
c) positive reinforcement
d) flooding
e) relationship therapy

25) The most frequently reported side effect of tricyclic


antidepressants is:
a) peripheral neuropathy
b) photosensitivity
c) agranulocytosis
d) jaundice
e) dry mouth
26) A high risk of suicide is associated with which one of the
following factors:
a) female aged less than 30 years
b) married male less than 30 years
c) public setting
d) secondary gain from attempt
e) single male aged more than 60 years
27) A fixed unalterable belief that is false in its content and in its
social and cultural setting is called:
a) an illusion
b) a hallucination
c) a delusion
d) agnosia
e) paranoia
28) Criteria for involuntary hospitalization of a suicidal patient
may include:
a) acute risk of harm to themself
b) inability to care for self
c) acute risk of harm to others
d) (b) and (c)
e) (a), (b), and (c)
29) Each of the following statements about affective disorders is true
EXCEPT:
a) patients with bipolar disorder show roughly the same
frequency of positive family history as do patients with
unipolar disorder
b) major depressive illness is more common in women than in
men
c) bipolar affective disorder is far less common than unipolar
affective disorder
d) there is a different response to lithium in unipolar and bipolar
disorder
e) imipramine is more likely to produce hypomania in bipolar
patients than in unipolar patients
30) A major depressive disorder is diagnosed only when the affective
disturbance has existed for at least :
a) two weeks
b) four weeks
c) six weeks
d) eight weeks
e) twelve weeks

22 Sample Questions

Psychiatry

31) A 30 year-old man presents in emergency with right lower quadrant


abdominal pain. His wife reports that he had been drinking heavily
in response to marital problems and had never had such pain
before. Appendicitis was diagnosed and an appendectomy was
successfully performed. Four days later the patient was anxious,
restless, unable to sleep and claimed his wife was a stranger trying
to harass him. The likeliest diagnosis is:
a) paranoid reaction
b) delirium tremens
c) mania
d) schizophreniform reaction
e) post-operative delerium
32) Which of the following symptoms of schizophrenia responds best to
anti-psychotic medication:
a) anti-social behavior
b) flat affect
c) paranoid delusions
d) lack of motivation
e) all of the above
33) A 53 year-old housewife presents with depression marked by early
morning wakening, diminished energy and poor concentration. She
is treated with supportive psychotherapy and celexa
20 mg. After four weeks she is no better. You would next:
a) reassure her that antidepressants take 5-6 weeks to be
effective
b) begin intensive psychotherapy
c) switch to a different SSRI
d) increase the dose to 40 mg
e) (a) and (d)
34) Hypertensive encephalopathy is a serious complication of treatment
with:
a) phenothiazines
b) tricyclic antidepressants
c) lithium carbonate
d) MAOI antidepressants
e) benzodiazepines
35) Electroconvulsive therapy (ECT) is a treatment in
which one of the following conditions?
a) obsessive compulsive disorder
b) paranoid schizophrenia
c) generalized anxiety disorder
d) acute mania
e) major depression with psychotic features
36) Tardive dyskinesia is:
a) an acute extrapyramidal side effect of antipsychotic
b) an acute anticholinergic side effect of tricyclic antidepressants
c) a type of Parkinsons disease
d) a side effect of excessive ECT treatments
e) a long term complication of chronic antipsychotic
administration
37) Which of the following symptoms is most commonly found in
schizophrenia?
a) depressed mood
b) flight of ideas
c) elevated mood
d) delusional fears
e) thought insertion
38) Patients with conversion disorders will show each of the following
EXCEPT:
a) la belle indifference
b) severe depression
c) loss of special sense function
d) secondary gains
e) paralysis of voluntary muscles
39) The syndrome of delirium tremens is associated with each of the
following EXCEPT:
a) rapidly fluctuating level of consciousness
b) dehydration
c) visual hallucinations
d) renal failure
e) polyneuropathy

Toronto Notes 2005

40) Which of the following is best treated with high dose


benzodiazepines:
a) schizophrenia, catatonic type
b) major depression
c) generalized anxiety disorder
d) delirium tremens
e) psychogenic amnesia
41) Personality types predisposed to depression include which
one of the following:
a) dependent
b) antisocial
c) schizoid
d) paranoid
e) schizotypal
42) Which of the following neurological symptoms can be produced
by antipsychotic drugs?
a) akathesia
b) shuffling gait
c) oculogyric crisis
d) tremor at rest
e) all of the above
43) Absolute contraindications to ECT include:
a) pregnancy
b) recent myocardial infarction
c) fractured pelvis
d) brain tumor
e) all of the above
44) In the elderly delirium may be produced by the use of:
a) neuroleptics
b) tricyclic antidepressants
c) antiparkinsonian agents
d) minor tranquilizers
e) all of the above
45) Which of the following statements about simple phobia is correct?
a) it responds well to simple reassurance
b) it responds well to individual psychotherapy
c) it responds well to benzodiazepines
d) it responds well to relaxation and desensitization
e) all of the above
46) Toxicity due to lithium carbonate is associated with all
of the following EXCEPT:
a) nausea
b) serum lithium level of 0.1 mEq/L
c) tremulousness
d) convulsions
47) Which of the following investigations need not be performed
before starting treatment with lithium carbonate?
a) serum creatinine
b) serum electrolytes
c) thyroid function studies
d) serum bilirubin
e) all of the above
48) All of the following are common side effects of tricyclic
antidepressants EXCEPT:
a) dry mouth
b) constipation
c) Parkinsonian tremor
d) tachycardia
49) Which of the following features are seen in dementia due to
cerebral arteriosclerosis:
a) patchy amnesia for remote events
b) more common in women than men
c) death occurs frequently due to cerebrovascular
accidents
d) increased senile plaque count
e) all of the above

Toronto Notes 2005

Psychiatry

50) The Theory of Behavioral Therapy employs which of the following


concepts:
a) reaction formation
b) reinforcement
c) imprinting
d) autosuggestion
e) sublimation
51) Which of the following represents a contraindication for
psychoanalysis?
a) the existence of transference feeling toward the analyst
b) the existence of countertransference feeling in the analyst
c) the resistance to change by the patient
d) preoccupation with resolving an acute crisis situation
e) the existence of an underlying neurotic style of
behaviour
52) Which of the following is least helpful in treating depression?
a) phenelzine
b) imipramine
c) lithium carbonate
d) lorazepam
e) all of the above

Sample Questions 23

53) A young mother is very focused on the health of her


16 month-old. She keeps her house immaculate for fear that
dirt will harm her baby, she checks the lock on the door at
least ten times before retiring to bed, and she has to get up
and check that her child is still breathing at least 3 times
every night. She knows that her fears are irrational but
persists with these behaviors. The most likely diagnosis is:
a) paranoid delusions not otherwise specified
b) post-partum depression
c) obsessive compulsive disorder
d) generalized anxiety disorder
e) paranoid personality disorder
54) A thorough assessment for the presence/absence of alcohol
withdrawal should include questions about all of the following
EXCEPT:
a) nausea and vomiting
b) mood
c) difficulty walking (ataxic gait)
d) visual disturbances
e) tremulousness
55) Which of the following will be least helpful with respect to
distinguishing delerium from dementia:
a) disorientation at night
b) duration of disorientation
c) fluctuating level of consciousness
d) mini mental status examination
e) presence of visual hallucinations

24 Sample Questions

Surgery

Toronto Notes 2005

Surgery
1) A 25 year-old known substance abuser is brought to the ED with a
suspected overdose. Which of the following is not considered a
universal antidote?
a) glucose
b) oxygen
c) calcium gluconate
d) naloxone
e) thiamine

10) Which of the following is not a sign or symptom of carpal tunnel


syndrome :
a) loss of sensation to the proximal palm
b) positive Phalen test
c) positive Tinel sign
d) pain involving the thumb, index, long, and part of the ring
digits
e) delayed transit time on nerve conduction study

2) Which of the following is not a classic sign of a basal skull fracture?


a) Battle sign
b) racoon eyes
c) hemotympanum
d) Freedman sign
e) CSF rhinnorhea/ottorrhea

11) A 35 year-old intoxicated man presents to the Emergency


Department with a deep laceration to his right distal forearm after
putting his hand through a window. On examination, he has no
sensation to his little finger and the ulnar aspect of his ring finger.
Power on abduction and adduction of all fingers is markedly
decreased. He is unable to flex the distal joint of his little finger.
When he flexes his wrist, his hand deviates radially. The
examination was otherwise normal. What structure(s) have been
damaged?
a) ulnar nerve
b) flexor carpi ulnaris and flexor digitorum profundus muscles
c) median nerve
d) radial nerve
e) brachioradialis

3) A 19 year-old female with a traumatic head injury is brought to the ED


by EMS. She is hemodynamically stable but requires assisted
ventilation through an endotracheal tube. She does not open her eyes
to painful stimuli and maintains an abnormal extension posture.
Which of the following is incorrect?
a) her GCS is 3T
b) a GCS of 8 is an indication for intubation
c) isolated head injuries can cause shock
d) she should be ventilated to a pCO2 of 30-35 mm Hg
e) all are incorrect
4) Conductive hearing loss is a symptom of:
a) presbycusis
b) Meniere disease
c) cholesteatoma
d) Bell palsy
e) acoustic neuroma
5)

The following statements regarding epistaxis are false EXCEPT:


a) Epistaxis rarely occurs in children
b) It commonly results from rupture of posterior placed nasal vessels
c) It may be treated by ligation of the ipsilateral internal carotid
artery
d) Epistaxis may be treated by cautery of Little's area with silver
nitrate
e) It is not a common cause of emergency admission to ENT wards

6) A 52 year-old dentist comes to your office complaining of severe


bilateral buttock cramps and thigh fatigue during a tennis match and
recent onset of impotence The most likely diagnosis is:
a) lumbosacral disc problem
b) multiple sclerosis
c) Leriche syndrome
d) metastatic carcinoma of the spine
7) A young woman who was involved in a cliff diving accident is
brought into the emergency department unresponsive to deep pain
with a right pupil that is dilated and nonreactive. The left pupil is
normal. The most appropriate inital treatment is:
a) endotracheal intubation
b) CT scan of the head
c) 500 cc normal saline IV
d) IV steroids
e) IV mannitol
8) Patients with an organic cause for impotence are often characterized
by all of the following EXCEPT:
a) diabetes
b) older age
c) intermittent difficulty
d) nocturnal penile tumescence absent
e) none of the above
9) In the investigation of infertility, a normal semen analysis MUST
contain:
a) a volume of 0.5-1 mL
b) morphology > 85% normal forms
c) WBC < 1 per high power field
d) motility > 75%
e) sperm count > 20 million sperm/mL

12) In which of the following are systemic antibiotics not indicated :


a) animal bite to hand
b) laceration over metacarpal-phalangeal joint sustained after
punching someone in a fight
c) positive Finkelstein test
d) burn wound to 75% of the dorsal aspect of the hand
e) acute suppurative tenosynovitis
13) Prolonged vomiting is associated with what electrolyte
abnormality?
a) hypochloremic hypokalemic metabolic acidosis
b) hypochloremic hypokalemic metabolic alkalosis
c) hyperchloremic metabolic acidosis
d) hyperkalemia
e) none of the above
14) Indications for immediate operative intervention rather than
conservative management for arterial insufficiency include all the
following EXCEPT:
a) nocturnal limb pain
b) ischemic ulceration
c) absent pulse
d) ischemic neuropathy
e) toe gangrene
15) Surgical indications for diverticulitis include all of the following
EXCEPT:
a) peritonitis
b) persisting hemorrhage
c) fistula
d) greater than two severe attacks
e) palpable abdominal mass in left lower quadrant
16) Six days following a hemicolectomy for colorectal carcinoma, a 54
year-old woman experiences calf pain unilaterally. On physical
examination, the affected side has a greater diameter than the
other, and there is tenderness to palpation. Which of the
following investigations would be the next step:
a) venogram
b) compression venous Doppler flow studies
c) x-ray of the affected calf
d) ventilation/perfusion scan
e) compartment pressure monitoring
17) All of the following radiologic abdominal plain film findings are
consistent with mechanical bowel obstruction except:
a) a "step ladder" pattern
b) dilated small bowel loops
c) air-fluid levels at uniform height in same bowel loop
d) absence of gas in large bowel
e) rows of small gas accumulations in valvulae conniventes (i.e.
"string of pearls")

Toronto Notes 2005

Surgery

18) After a weekend of heavy drinking, a 28 year-old male presents with


abdominal pain radiating to the back. What is the investigative and
prognostic modality of choice for the suspected diagnosis?
a) abdominal ultrasound
b) abdominal CT
c) abdominal plain film
d) endoscopic retrograde cholangiopancreatography
e) laparoscopy
19) An 18 year-old motorcyclist presents in the emergency department
following an accident. He has a compound tibia and fibula fracture
of the right leg and on examination the right leg has no pulses. Your
immediate treatment should be:
a) immediate angiogram
b) immediate surgery
c) casting and/or splinting
d) reduction and splinting
e) x-ray
20) Which of the following is the most serious complication of a dis
placed supracondylar fracture of the humerus?
a) compartment syndrome of the forearm
b) failure to heal
c) healing in a non-anatomical position
d) injury to the median nerve
e) significantly limited range of elbow motion
21) What is the typical audiogram finding in noise induced hearing loss:
a) conductive loss in the low frequencies
b) conductive loss at 2000 Hz
c) sensorineural loss at 2000 Hz
d) sensorineural loss at 4000 Hz
e) mixed broad spectrum hearing loss
22) Which of the following is not a complication of untreated otitis
media?
a) cholesteatoma
b) meningitis
c) tympanic membrane perforation
d) trigeminal neuralgia
e) conductive hearing loss
23) With respect to control of micturition, all of the following are true
EXCEPT:
a) damage to the cerebral cortex results in hyperactivity of the
detrusor
b) the basal ganglia inhibits bladder overactivity
c) the cerebellum coordinates emptying of the bladder
d) damage to the cerebellum results in hypotonicity of the detrusor
e) injury above the brainstem results in hyperactive bladder only
24) Which of the following drugs will not promote urine retention?
a) sympathomimetics
b) anticholinergics
c) cholinergic agonists
d) TCAs
e) smooth muscle depressants
25) A 28 year-old male presents with painless swelling in the testicle.
On examination, it appears to be a rubbery, hard mass. He has no
palpable abdominal masses. A testicular ultrasound confirms the
diagnosis of testicular tumour, and an abdominal CT reveals normal
retroperitoneal nodes. Which statement is correct with respect to this
case?
a) his overall prognosis is very poor
b) the testicular mass is likely a secondary tumour
c) a needle aspiration of the tumour is indicated
d) a transcrotal approach for biopsy is contraindicated
e) he will likely require chemotherapy
26) A 78 year-old woman complains of experiencing headaches and
progressive confusion for the last month. She has a left hemianopia
and cannot dress herself. A CT scan demonstrates a large, irregularly
enhancing mass in the right parietal lobe. There is no obvious
systemic disease. The most likely diagnosis is:
a) brain abscess
b) glioblastoma multiforme
c) meningioma
d) metastasis
e) CNS lymphoma

Sample Questions 25

27) A patient with a subarachnoid hemorrhage (SAH) caused by a right


anterior communicating artery aneurysm undergoes successful
surgery 2 days after the hemorrhage. Three days later, right arm
weakness develops. The most likely diagnosis is:
a) hydrocephalus
b) meningitis
c) repeat hemorrhage
d) vasospasm
e) none of the above
28) Complications of tonsillectomy include all of the following EXCEPT:
a) secondary hemorrhage
b) severe otalgia
c) Quinsy
d) nasopharyngeal stenosis
e) none of the above
29) Features characteristic of acute tonsilitis include all of the following
EXCEPT:
a) odynophagia
b) cough
c) referred otalgia
d) cervical lymphadenopathy
e) dysphagia
30) A 6 month-old baby has had mild inspiratory stridor for the last
2 months. The most likely cause of this is:
a) laryngomalacia
b) acute epiglottitis
c) croup
d) tonsillar hypertrophy
e) foreign body aspiration
31) A 48 year-old male is brought to the ED with CPR being
administered. The ECG shows electrical activity present but you
cannot palpate a carotid pulse. Which of the following is not on the
differential for pulseless electrical activity?
a) hypotension
b) hypokalemia
c) cardiac tamponade
d) hypothermia
e) tension pneumothorax
32) A 37 year-old male arrives at the Emergency Department
unconscious. He is warm and sweaty. His heart rate is 52 bpm, his
BPis 90/60. His pupils are constricted, his eyes are teary, and he is
drooling. You assume he is suffering from a toxidrome. What
antidote will you give him?
a) Flumazenil
b) Naloxone
c) Glucagon
d) Atropine
e) Ethanol
33) A 60 year-old man presents with back and leg pain and trouble
urinating. On examination, he has decreased sensation over the
buttocks, normal motor power, and absent ankle jerks bilaterally.
How would this patient best be investigated?
a) plain films of the lumbar spine
b) CT of lumbar spine
c) MRI of spine
d) investigate only if no improvement after 6 weeks of symptomatic
treatment
e) gallium scan of the spine
34) A 34 year-old woman presents with a 3 day history of increasing
fever with chills, sweats, nausea, and 2 episodes of vomiting. On
examination, she has costovertebral angle tenderness and urine
reveals gross pyuria. Initial management should include:
a) abdominal ultrasound
b) abdominal CT
c) empirical treatment with antibiotics; image only if she fails to
respond to therapy
d) treat with antibiotics and perform IVP 4-6 weeks after resolution
of illness
e) immediate DTPA renal scintigraphy

26 Sample Questions

Surgery

35) All of the following statements regarding knee injuries are correct
EXCEPT:
a) locking of the knee may be due to a torn meniscus
b) minor tears of the medial collateral ligament can be treated
with brief immobilization then range of motion and
strengthening exercises
c) Lateral meniscus tears are more common than medial meniscal
tears
d) anterior cruciate ligament tears may give a positive Lachman
test
e) a knee dislocation may be associated with major ligament
damage
36) An 83 year-old man has fallen while walking down stairs. He is
brought to the emergency department with a 3-part
intertrochanteric hip fracture. Which of the following procedures
would you choose to perform?
a) hemiarthoplasty
b) total hip replacement
c) multiple pin fixation
d) bipolar arthroplasty
e) pin and plate
37) A 45 year-old man with a history of polycystic kidney disease
presents with painless gross hematuria. You order:
a) no investigation is required since the hematuria is most likely
due to the rupture of renal cyst(s)
b) no investigation at this time. Investigate if gross hematuria
persists
c) U/S
d) U/S, urine C&S, cystoscopy
38) A 75 year-old man with a history of nocturia has not micturated
for the last 10 hours, and is complaining of severe lower abdominal
pain. The most likely cause is:
a) BPH
b) prostate cancer
c) renal failure
d) UTI
e) prostatitis
39) A 43 year-old woman is brought to the Emergency Department
after being burned in a house fire. You estimate first degree burns
to 20% of her body, second degree burns to 11% of her body, and
third degree burns to 9% of her body. She weighs 60 kg and is
120 cm tall. What IV therapy would you begin immediately?
a) normal saline at 200 cc/hr for 24 hours
b) normal saline at 400 cc/hr for 24 hours
c) normal saline at 150 cc/hr for 8 hours, then 75 cc/hr for the
next 16 hours
d) normal saline at 300 cc/hr for 8 hours, then 150 cc/hr for the
next 16 hours
e) normal saline at 600 cc/hr for 8 hours, then 300 cc/hr for the
next 16 hours
40) Which of the following statements is true?
a) skin grafts have their own blood supply
b) granulation tissue, bone, and perichondrion can all support a
skin graft
c) wound contracture does not occur with skin grafting
d) axial flaps do not rely on a particular anatomically defined
vascular bundle
e) muscle flaps can aid in immunologic defense in infected
wounds
41) Risk factors for hepatocellular carcinoma include all of the
following EXCEPT:
a) hepatitis A
b) cirrhosis
c) exogenous steroid use
d) hemochromatosis
e) smoking
42) Which of the following is associated with biliary colic?
a) epigastric pain
b) rebound tenderness
c) jaundice
d) Murphy sign
e) all of the above

Toronto Notes 2005

43) Which of the following is not a surgically correctable cause of


hypertension?
a) aortic coarctition
b) renal artery atherosclerosis
c) pheochromocytoma
d) primary hyperaldosteronism (Conn syndrome)
e) none of the above
44) A 16 year-old female fell while roller-blading on her outstretched
right hand. At a nearby emergency department X-rays confirmed
the diagnosis of a closed Colles fracture. The proper reduction
technique for this wrist fracture is which of the following:
a) slight extension, full pronation, and full ulnar deviation
b) slight flexion, full supination, and full radial deviation
c) slight extension, full supination, and full ulnar deviation
d) slight flexion, full pronation, and full ulnar deviation
e) slight extension, full pronation, and full radial deviation
45) Which of the following is least likely to cause avascular necrosis:
a) sickle cell disease
b) septic arthritis
c) steroid use
d) constrictive dressings
e) post-traumatic fracture
46) A 24 year-old football player severely fractures his ankle while
playing in a game and subsequently requires ORIF treatment.
Indications for ORIF treatment of an ankle fracture include all of
the following except:
a) a fracture-dislocation
b) undisplaced fracture with Grade II ATFL tear
c) trimalleolar fracture
d) unstable talar tilt
e) unable to maintain a closed reduction
47) Which of the following is not an indication for repair of an orbital
blow-out fracture?
a) enopthalmos
b) persistent diplopia
c) positive forced duction test
d) orbital floor lesion < 0.7 cm
e) other upper facial fractures
48) Which of the following is true regarding mandibular fractures?
a) they are predominantly unilateral
b) compound fractures are rarely intraoral
c) they may manifest with numbness in the V2 facial nerve
distribution
d) malocclussion of teeth is a common sign
e) they seldom result in trismus
49) With respect to painful scrotal swelling, all of the following are
true EXCEPT:
a) torsion of testicular appendages will usually subside without
surgical intervention
b) nausea/vomiting is very common in epididymitis
c) ultrasound is helpful in determining the cause of hematocele
d) torsion most commonly occurs in young males
e) orchitis may result in testicular atrophy
50) Which is following statement about PSA is true?
a) every man past the age of 50 should have an annual PSA test
as a primary screen for prostate cancer
b) PSA is best used to follow disease progression or recurrence
post-operatively
c) PSA is a useless test
d) increased complex PSA to total PSA ratio favors BPH over
prostate cancer
e) all of the above
51) A 40 year-old women presents to the ER with fever, no nausea or
vomiting, and left flank pain radiating to the groin. Body CT
reveals 8 mm stone in the left proximal ureter. You should:
a) send her home because renal stone is benign disease
b) send her home with oral analgesics and ask her to come back if
the pain does not resolve in the next 24 hours
c) admit, close monitoring, hydration and analgesics
d) admit, hydration, analgesics and IV antibiotics. (e.g. Amp and
Gent), and consider stenting if the symptom persists

Toronto Notes 2005

Surgery

52) Nasopharyngeal carcinoma:


a) is most common in people from south-east Africa
b) presents early with nasal pain
c) is treated by wide surgical excision
d) is monitored by measurement of Epstein-Barr virus antibodies
e) is curable in 90% of patients
53) Factors which contribute to the development of squamous cell
cancers of the head and neck include all of the following EXCEPT:
a) cigarette smoking
b) alcohol ingestion
c) aging
d) exposure to particular emission from diesel engines
e) solar irradiation
54) Which of the following statements about aspiration of a peanut
into the tracheobronchial tree is FALSE?
a) the peanut is more likely to be in the left lower lobe bronchus.
b) expiratory wheeze is the most likely finding on physical exam
c) the peanut should be removed with the patient under general
anesthesia through an open bronchoscope with forceps
designed to grasp peanuts
d) a chest x-ray may show atelectasis distal to the blocked
bronchus
e) pneumonia may be a complication
55) Which of the following is an absolute indication for a
tonsillectomy?
a) airway obstruction
b) recurrent (>5) episodes of tonsillitis
c) peritonsillar abscess
d) tonsillar hypertrophy
e) halitosis
56) A 72 year-old man on physical examination is found to have
expressive dysphasia and mild right arm weakness. The most
probable location of his lesion is:
a) right parietal lobe
b) left frontal lobe
c) right frontal lobe
d) left parietal lobe
e) basal ganglia
57) Inital management of any patient with coma of undetermined
cause includes all except:
a) clear and secure the airway
b) naloxone
c) D50W 50 mL IV
d) dexamethasone 16 mg IV
e) thiamine
58) A 53 year-old male presents to the ED with new onset of a severe
headache associated with nausea and vomiting. There is no
history of trauma. He is alert and oriented with no neck stiffness.
Anisocoria is present. The most likely diagnosis is:
a) intracranial bleed
b) cluster headache
c) meningitis
d) migraine headache
e) tension headache
59) A 16 year-old girl is brought to hospital by her frantic parents after
a bee sting. Vitals sings are BP 70/40, RR 30 and laboured, HR 140,
T 37.5. Which of the following would not be an option in her
management?
a) epinephrine
b) diphenhydramine
c) methylprednisolone
d) salbutomol
e) atropine

Sample Questions 27

60) A 24 year-old woman arrives at the Emergency Department


unconscious. Her BP is 90/60, her heart rate is 60 bpm, she is
breathing at 8 breaths per minute and her O2 sat is 86%. Her
eyes remain closed even after pain stimulation and the only sounds
she makes are incomprehensible. Her elbows and wrists are flexed
with her feet extended. This patients GCS score is:
a) 3
b) 4
c) 5
d) 6
e) 7
61) Which physical exam finding below is usually not associated with
increasing intracranial pressure?
a) deteriorating level of consciousness
b) increasing heart rate and increasing blood pressure
c) yawning, hiccuping, vomiting
d) seizures
e) unilateral sixth cranial nerve palsy
62) Which of the following is false regarding post operative wound
infections?
a) S. aureus is the most common cause
b) usually present with fever post of day 3-4
c) increased likelihood in diabetics
d) mainly treated with antibiotics
e) risk increases with length of surgery
63) Which of the following signs and symptoms warrant surgical
intervention for patients with small bowel obstruction?
a) abdominal tenderness
b) air-fluid levels on abdominal x-ray
c) worsening abdominal pain
d) feculent vomitus
e) air in the colon and rectum on abdominal x-ray
64) A 59 year-old woman presents to her family physician with a 3 cm
palpable, well circumscribed, non-tender breast mass. She first
noticed it several months ago and believes it has increased in size
since then. Her mother and maternal aunt were diagnosed with
breast cancer in their early 50's. She has no other health complaints.
A subsequent mammogram shows no abnormalities. Which of the
following is the next most appropriate step in management?
a) prophylactic mastectomy
b) repeat mammogram in 6 months
c) repeat mammogram ion 1 year
d) excisional biopsy
e) ultrasound
65) Which of the following is false regarding pancreatic pseudocysts?
a) it is caused by duct leakage
b) clinically suspected if persisting pain > 2 weeks following
diagnosis of acute pancreatitis
c) majority are treated surgically
d) surgical intervention is typically delayed to allow pseudocyst to
mature
e) lacks true epithelium
66) An overweight, 45 year-old man presents with left lower quadrant
tenderness and a one week history of abdominal pain, loose nonbloody stools and worsening fever. Laboratory investigation yields
leukocytosis with neutrophilia and left shift. What is the diagnostic
modality of choice?
a) CT scan
b) barium enema
c) ultrasound
d) plain abdominal film
e) angiography
67) A 19 year-old woman is brought to the emergency room following
involvement in a motor vehicle accident. On examination, she has a
GCS of 10 and swelling over the occipital protuberance. The most
appropriate imaging study is:
a) MRI of skull and contents
b) skull films
c) head CT with contrast
d) head CT without contrast
e) CT myelogram

28 Sample Questions

Surgery

68) An 80 year-old woman suffers cervical spinal soft-tissue injury in a


motor vehicle accident with no skeletal or neurologic damage
documented at the time. Three months later, she presents with
sudden onset of homonymous right upper quadrantanopia. CT
demonstrates a non-hemorrhagic lesion in the left lower occipital
lobe. Which imaging study would likely yield the most useful
information?
a) carotid Doppler ultrasound
b) echocardiography
c) MR angiography
d) SPECT scan
e) C-spine plain films
69) Which of the following radiographic features is most consistent
with osteoarthritis of the knee?
a) marginal erosions
b) juxta-articular osteopenia (demineralization)
c) loss of articular cartilage with narrowing of the radiologic joint
space
d) osteonecrosis (avascular necrosis) of the medial femoral condyle
e) syndesmophyte formation
70) Which of the following statements is incorrect with respect to
wound healing?
a) epithelialization can occur within 24 hours following primary
closure of a wound
b) maximum wound strength is often achieved after 2 years
c) wounds continue to gain strength after collagen synthesis has
reached an equilibrium
d) wound contracture is mediated by myofibroblasts
e) the incidence of wound infection increases with healing by
secondary intention
71) Which of the following is not a cause of sensorineural hearing loss:
a) ossicular discontinuity
b) ototoxicity
c) Mnire disease
d) noise
e) presbycusis
72) Which of the following is not true of Menieres disease?
a) characterized by quadrad of vertigo, hearing loss, tinnitus and
aural fullness
b) vertigo burns out with time
c) can be treated with diuretics
d) it is the result of an abnormal buildup of potassium
e) the vertigo lasts for seconds
73) A neurosurgeon complains of a 3 week history of awakening at
night with right-hand discomfort that resolves after several minutes.
On examination, he has mild weakness of thumb abduction and
diminished pain sensibility on the palmar aspect of the thumb and
index finger. The most likely diagnosis is:
a) carpal tunnel syndrome
b) cervical radiculopathy
c) reflex sympathetic dystrophy
d) tendonitis
e) left middle cerebral artery ischemic attacks
74) A 73 year-old woman presents with a 6 month history of
deteriorating gait and low back discomfort, exacerbated by walking.
Examination is unremarkable except for hypoactive muscle stretch
reflexes in the legs. X-rays of the lumbosacral area shows the
expected degenerative changes associated with a woman of her age.
The most likely diagnosis is:
a) acute lumbar disc hernation
b) lumbar stenosis
c) myopathy
d) normal pressure hydrocephalus
e) cervical stenosis

Toronto Notes 2005

75) Which of the following findings is inconsistent with cardiac


tamponade?
a) hypotension
b) pulsus paradoxus
c) Kussmaul sign
d) jugular venous distension
e) muffled heart sounds
76) Tension pneumothorax is best diagnosed with:
a) stat CT scan
b) chest x-ray
c) watch and wait
d) clinical exam
e) none of the above
77) Which of the following is NOT a cause of major lower
gastrointestinal hemorrhage?
a) diverticulitis
b) angiodysplasia
c) aortoenteric fistula
d) none of the above
78) All of the following have been associated with posterior shoulder
dislocation EXCEPT:
a) ethanol
b) electricity
c) exercise
d) epilepsy
e) encephalitis
79) Management of an open fracture should always include each of the
following EXCEPT:
a) assessment of neurovascular status
b) reduction and fixation of fracture
c) irrigation and debridement of wound
d) application of sterile dressing
e) application of topical antibiotic
80) What is the most common type of thyroid cancer?
a) medullary
b) papillary
c) follicular
d) lymphoma
e) anaplastic

Toronto Notes 2005

Answers to Sample Questions

Sample Questions 29

Answers to Sample Questions


COMMUNITY AND PUBLIC HEALTH
1.
2.
3.
4.
5.
6.
7.
8.
9.
10.
11.
12.
13.

D
D
C
B
C
C
E
E
D
B
A
B
C

14.
15.
16.
17.
18.
19.
20.
21.
22.
23.
24.
25.
26.

E
D
B
D
A
D
A
E
B
C
E
D
E

27.
28.
29.
30.
31.
32.
33.
34.
35.
36.
37.
38.
39.

D
E
D
B
D
B
D
B
D
B
C
A
B

40.
41.
42.
43.
44.
45.
46.
47.
48.
49.
50.
51.
52.

A
C
D
C
B
A
E
B
C
E
C
D
B

53. C

16.
17.
18.
19.
20.
21.
22.
23.
24.
25.
26.
27.
28.
29.
30.

C
B
E
D
E
E
D
E
D
B
C
A
D
C
C

31.
32.
33.
34.
35.
36.
37.
38.
39.
40.
41.
42.
43.
44.
45.

B
E
D
C
C
B
F
C
A
B
B
A
E
D
D

46.
47.
48.
49.
50.
51
52.
53.
54.
55.
56.
57.
58.
59.
60.

A
D
D
D
D
D
D
B
A
B
C
E
A
B
E

61.
62.
63.
64.
65.
66.
67.
68.
69.
70.
71.
72.
73.
74.
75.

E
A
C
D
E
C
B
D
D
D
C
A

37.
38.
39.
40.
41.
42.
43.
44.
45.
46.
47.
48.

C
B
C
E
B
D
B
B
B
D
B
A

49. E
50. D

MEDICINE
1.
2.
3.
4.
5.
6.
7.
8.
9.
10.
11.
12.
13.
14.
15.

C
B
D
A
E
C
C
E
D
B
D
B
B
A
C

C
D
D
B
D
E
C
B
C
D
B
B
E
B
C

OBSTETRICS AND GYNECOLOGY


1.
2.
3.
4.
5.
6.
7.
8.
9.
10.
11.
12.

C
B
D
C
D
C
E
C
E
B
E
B

13.
14.
15.
16.
17.
18.
19.
20.
21.
22.
23.
24.

A
E
C
C
D
B
B
C
E
C
B
E

25.
26.
27.
28.
29.
30.
31.
32.
33.
34.
35.
36.

76. E
77. D
78. C

30 Sample Questions

Answers to Sample Questions

Toronto Notes 2005

PEDIATRICS
1.
2.
3.
4.
5.
6.
7.
8.
9.
10.
11.
12.
13.
14.
15.
16.

E
A
A
C
A
C
B
E
D
A
B
E
D
B
A
B

17.
18.
19.
20.
21.
22.
23.
24.
25.
26.
27.
28.
29.
30.
31.
32.

B
B
B
B
A
A
E
B
D
C
B
C
A
B
B
E

33.
34.
35.
36.
37.
38.
39.
40.
41.
42.
43.
44.
45.
46.
47.
48.

C
C
D
A
C
C
A
C
E
A
B
D
C
A
B
B

49.
50.
51.
52.
53.
54.
55.
56.
57.
58.
59.
60.
61.
62.
63.
64.

E
D
A
B
D
E
A
E
D
C
C
C
D
E
A
E

15.
16.
17.
18.
19.
20.
21.
22.
23.
24.
25.
26.
27.
28.

E
B
D
A
D
E
D
D
C
C
E
E
C
E

29.
30.
31.
32.
33.
34.
35.
36.
37.
38.
39.
40.
41.
42.

A
A
B
C
E
D
E
E
D
B
D
D
A
E

43.
44.
45.
46.
47.
48.
49.
50.
51.
52.
53.
54.
55.

D
E
D
B
D
C
C
B
D
D
C
C
A

15.
16.
17.
18.
19.
20.
21.
22.
23.
24.
25.
26.
27.
28.

E
B
C
B
D
A
D
D
D
C
D
B
D
C

29.
30.
31.
32.
33.
34.
35.
36.
37.
38.
39.
40.
41.
42.

B
A
B
D
C
C
C
E
D
A
D
E
A
A

43.
44.
45.
46.
47.
48.
49.
50.
51.
52.
53.
54.
55.
56.

D
D
B
D
D
D
B
B
D
D
D
A.
A
B

65.
66.
67.
68.
69.
70.
71.
72.
73.
74.
75.
76.
77.
78.
79.
80.

B
E
C
E
B
A
B
D
A
A
B
B
C
B
D
D

81. B
82. D
83. E

57.
58.
59.
60.
61.
62.
63.
64.
65.
66.
67.
68.
69.
70.

D
A
E
D
B
D
C
D
C
A
D
C
C
E

71.
72.
73.
74.
75.
76.
77.
78.
79.
80.

PSYCHIATRY
1.
2.
3.
4.
5.
6.
7.
8.
9.
10.
11.
12.
13.
14.

D
A
D
C
E
E
B
B
B
D
B
E
D
C

SURGERY
1.
2.
3.
4.
5.
6.
7.
8.
9.
10
11.
12.
13.
14.

C
D
C
C
D
C
A
C
E
A
A
D
B
C

A
E
A
B
C
D
A
C
E
B

Toronto Notes 2005

Key Feature Questions

Sample Questions 31

Key Feature Questions


Instructions for Examinees
Key Feature questions can pertain to solutions of clinical cases that involve data-gathering (e.g., history taking, physical examination, laboratory
investigations), diagnosis or treatment.
The Key Feature questions will ask for your answers in different ways: you may be asked to (i) select the correct response(s) from a menu of options, or
(ii) print the correct answer(s) on a blank line.
Please note the following guidelines before proceeding:
1.

After reading the description of the case, read the question carefully. Take note as to whether you are told the maximum number of answers to
provide. If you exceed this number, you will receive no credit for the question. If you select or list fewer answers, you will still receive credit
for any of your answers which are correct.

2.

If you are asked to select your answers from a menu:


a. Read the complete menu of options before recording your answers.
b. Make certain that the number of answers you record is not greater than the number you have been asked to select.

3.

If you are asked to list your answers below the question:


a. Ensure that you print each answer legibly.
b. Record each answer on a separate blank line.
c. Do not record more than the maximum number you are asked to list.

CASE1
A 30 year-old homeless male presents with a 4 week history of feeling
generally unwell with anorexia, weight loss and intermittent fever. He
lives primarily on the streets or occasionally in a mens shelter. He
admits to regular use of intravenous heroin for the last seven years and
often shares needles.

QUESTION 1 (CASE 1)

QUESTION 2 (CASE 1)

Given this history, which of the following diagnoses would you consider?
Select up to five.
1. Acute pancreatitis
2. AIDS related complex
3. Aspergillosis
4. Bacterial endocarditis
5. Chronic pancreatitis
6. Delirium tremens
7. Diabetes mellitus
8. Hepatitis B
9. Hepatocellular carcinoma
10. Histoplasmosis
11. Leukemia
12. Lymphoma
13. Osteomyelitis
14. Peptic ulcer disease
15. Pericarditis
16. Pyelonephritis
17. Renal failure
18. Secondary syphilis
19. Talc pneumonitis
20. Tuberculosis

A mitral regurgitant murmur is auscultated in this patient.


His temperature is 39.0C.
Which of the following other findings may be found on physical
examination given the diagnoses being considered?
Select up to five.
1. Ascites
2. Campbell De Morgans spots
3. Cannon a waves
4. Cyanosis
5. Erythema nodosum
6. Follicular keratosis
7. Gynecomastia
8. Hepatomegaly
9. Janeway lesions
10. Jaundice
11. Koilonychia
12. Livedo reticularis
13. Osler nodes
14. Pallor
15. Palmar erythema
16. Papilledema
17. Purpura
18. Roth spots
19. Spider nevi
20. Splenomegaly

32 Sample Questions

Key Feature Questions

Toronto Notes 2005

CASE 2

CASE 3

A babysitter brings a 14 month-old to emergency because of inconsolable crying. The child appears well cared for and healthy but a
"bruise" is noted on the left humerus and a well-circumscribed small
fresh burn is noted on the other shoulder.

An 84 year-old female is brought to the Emergency Department by her


daughter who states that her mother has not been herself over the past
three days. Further questioning reveals that the mother has been less
attentive than usual and more withdrawn, her conversation is often
rambling and she has not been sleeping well. Prior to this, she functioned very well.

QUESTION 3 (CASE 2)
List four well recognized risk factors for child abuse:
1. _____________________________________________________________
2. _____________________________________________________________
3. _____________________________________________________________
4. _____________________________________________________________

QUESTION 4 (CASE 2)
Which of these physical findings are most suggestive of child abuse?
Select up to four.
1. Alopecia
2. Blue sclera
3. Bruises on shins and elbows
4. Buttocks wasting
5. Caput medusa
6. Clubbing
7. Condylomata acuminata
8. Eczema
9. Full fontanelle
10. Healed laceration on chin
11. Hutchison teeth
12. Impetigo
13. Large purple macular lesion on buttock
14. Limp
15. Oral thrush
16. Petechiae
17. Pitted finger nails
18. Proptosis
19. Retinal hemorrhage
20. Seborrhea

QUESTION 5 (CASE 2)
You suspect this is a case of child abuse. Which of the following are
appropriate steps in the initial management. Select up to four.
1. Abdominal ultrasound
2. Ascorbic acid level
3. Bone density studies
4. CBC
5. Chest x-ray
6. Coagulation studies
7. Creatinine
8. Dietary history
9. ECG
10. Factor 8 assay
11. Liver spleen scan
12. MRI
13. Notify CAS (Childrens Aid Society)
14. Notify police
15. Notify public health nurse
16. Ophthalmology consult
17. Psychiatry consult
18. Serum calcium, phosphorus and alkaline phosphorus
19. Skeletal survey
20. Urinalysis

QUESTION 6 (CASE 3)
What diagnoses would you consider at this time? List up to two.
1. _____________________________________________________________
2. _____________________________________________________________

QUESTION 7 (CASE 3)
What additional aspects of history would be most important to know?
Select up to four.
1. Alcohol use
2. Complete psychiatric history
3. Family history of Alzheimer disease
4. Financial status
5. History of fever
6. History of incontinence
7. History of osteoarthritis
8. History of Parkinson disease
9. History of previous CVA
10. Immunization status
11. Marital status
12. Medication history
13. Occupational history
14. Pattern of symptoms over the course of the day, e.g. fluctuations
15. Presence of hallucinations
16. Previous history of depression
17. Previous seizure disorder
18. Previous surgery
19. Recent travel
20. Smoking history

QUESTION 8 (CASE 3)
The interview confirms the daughter's description of the mother.
Physical exam reveals: BP 120/80 supine and 100/70 sitting, RR=22,
P=104, T=38.7C. JVP is at the sternal angle. Respiratory exam reveals
dullness, increased tactile fremitus, crackles and bronchial breath
sounds all in the left base. Heart sounds are normal. The abdominal
exam is normal. CNS exam does not reveal any focal findings. What
would you include in your initial investigations? Select up to four.
1. ALT
2. AST
3. Blood cultures
4. B12
5. CBC
6. Chest x-ray
7. CPK
8. CT scan of head
9. Drug screen
10. EEG
11. Electrolytes
12. HIV test
13. MRI of head
14. Protein electrophoresis
15. PT and PTT
16. RBC folate
17. SPECT scan
18. TSH
19. Urea and creatinine
20. VDRL

Toronto Notes 2005

Key Feature Questions

Sample Questions 33

CASE 4

CASE 5

A 57 year-old man presents to the Emergency Department with a 12


hour history of left flank pain.

A 30 year-old woman comes to your office with a 3 day history of vulvar and vaginal pruritus.

QUESTION 9 (CASE 4)

QUESTION 12 (CASE 5)

What diagnoses would you consider at this time?


Select up to four.
1. Ruptured abdominal aortic aneurysm
2. Bowel obstruction
3. Appendicitis
4. Pyelonephritis
5. Diverticulitis
6. Renal calculi
7. Biliary colic
8. Renal cell carcinoma
9. Pneumonia

What diagnosis would you consider at this time? List up to three.

QUESTION 10 (CASE 4)
On history and physical exam, what symptoms and signs might aid in
diagnosis? Choose up to seven.
1. Fever/chills
2. Weight loss
3. Rebound tenderness
4. Costovertebral angle tenderness
5. Urgency and frequency
6. Pruritus
7. Hypotension
8. Pulsatile abdominal mass
9 Nausea and vomiting
10. Left flank mass
11. Productive cough
12. Abdominal guarding
13. Pale stools and dark urine
14. Hematuria

QUESTION 11 (CASE 4)
What tests might be necessary to confirm your diagnosis?
Choose up to six.
1. ESR
2. Serum amylase
3. Urine R&M
4. Chest x-ray
5. Abdominal/pelvic ultrasound
6. 3 views of the abdomen
7. Urine C&S
8. Abdominal CT
9. IVP
10. Total bilirubin
11. Electrolytes
12. 24 hour creatinine clearance
13. KUB
14. Serum Mg
15. Serum alkaline phosphatase
16. Barium enema
17. Serum Ca
18. CBC

1. _____________________________________________________________
2. _____________________________________________________________
3. _____________________________________________________________

QUESTION 13 (CASE 5)
What features on the history would you inquire about to make a diagnosis? Choose up to seven.
1. Date of last menstrual period
2. Sexual history
3. Vaginal discharge
4. Vulvar erythema
5. Vaginal odor
6. Recent antibiotic therapy
7. Diabetes
8. Macular rash
9. Psychiatric history
10. Headache
11. Allergies
12. Dysuria
13. Use of douches
14. Sore throat
15. Previous history of STDs

QUESTION 14 (CASE 5)
What tests would you do to confirm the diagnosis? Choose up to four.
1. Urine C&S
2. Wet prep of vaginal discharge for microscopy
3. Urinalysis
4. CBC
5. ESR
6. Culture for gonorrhea
7. Culture for chlamydia
8. Abdominal CT
9. Pelvic ultrasound
10. Culture for Trichomonas vaginalis

34 Sample Questions

Key Feature Questions

Toronto Notes 2005

CASE 6

QUESTION 18 (CASE 7)

A 65 year-old man in the emergency department has RUQ pain. He is


otherwise healthy.

What questions would you ask on history to help determine the cause
of the bleeding? Choose up to five.
1. Family history of inflammatory bowel disease
2. Hematemesis
3. Recent weight loss
4. Nausea and vomiting
5. Change in bowel habits
6. Abdominal pain
7. Fever/chills
8. Family history of colo-rectal cancer
9. Blood mixed with stool or coating the surface
10. Past history of hemorrhoids
11. Dietary history
12. Smoking history
13. Pain on defecation
14. Allergies

QUESTION 15 (CASE 6)
List three diagnoses you should consider in this patient.
1. _____________________________________________________________
2. _____________________________________________________________
3. _____________________________________________________________

QUESTION 16 (CASE 6)
What are the most important questions to ask about on history?
Choose up to seven.
1. Pruritus
2. Pale stools
3. Fever/chills
4. Hematuria
5. Jaundice
6. Anorexia
7. Dark urine
8. Weight loss
9. Back pain
10. Diarrhea
11. Alcohol/drug history
12. Nausea and vomiting
13. Fatigue
14. Cold intolerance

CASE 8
You are called to the nursery to assess a 2 hour old term newborn with
respiratory distress. The only information the nurse gives you over the
phone is that the baby was born this morning, had good Apgars and
arrived in the nursery pink on room air and in no distress.

QUESTION 19 (CASE 8)
What are the most common diagnoses that would be in your differential for this infant? List up to three.
1. _____________________________________________________________
2. _____________________________________________________________
3. _____________________________________________________________

CASE 7
A 60 year-old woman comes to your office because she has seen bright
red blood in the toilet bowl after bowel movements for the past month.
She is otherwise healthy.

QUESTION 17 (CASE 7)
What are three common diagnoses you should consider?
1. _____________________________________________________________
2. _____________________________________________________________
3. _____________________________________________________________

QUESTION 20 (CASE 8)
With respect to the diagnosis, which elements of the history and physical will be most important in determining the etiology of this babys
distress? Select up to four.
1. Delivery mode (C-section, vaginal)
2. Feeding pattern
3. Grunting
4. Head circumference
5. Heart sounds
6. History of maternal diabetes
7. Indrawing
8. Maternal temperature
9. Meconium stained amniotic fluid
10. Oxygen requirement
11. Presence of bruising
12. Presenting part (breech, vertex)
13. Previous obstetric history
14. Tachypnea
15. Vital signs

Toronto Notes 2005

Key Feature Questions

Sample Questions 35

CASE 9

QUESTION 22 (CASE 9)

A 45 year-old migrant farm worker comes to see you with sudden


onset of severe pain in his back and right leg after lifting some bales of
hay. The pain radiates into the lateral aspect of the right foot and is
associated with numbness along the lateral aspect of the foot. He has
had a 5 year history of lower back pain with a similar episode of pain 1
year ago which was concentrated at the posterolateral aspect of the
right calf and lateral aspect of the right foot. It became progressively
worse, then resolved over a 3 week period.

What elements of the physical exam would you focus on?


Choose up to four.
1. Examination of skin
2. Range of motion of lumbar spine
3. Blood pressure
4. Inspection of muscle bulk in lower extremities
5. Sensory exam of arms
6. Peripheral pulses
7. Reflexes in lower extremities
8. Motor testing of legs
9. Motor testing of arms
10. Sensory exam of legs
11. Bowstring sign
12. Respiratory rate
13. Temperature
14. Straight leg raise test
15. Abdominal exam
16. Rectal exam
17. Pulse
18. Range of motion of knee
19. Auscultation of chest
20. Romberg test

QUESTION 21 (CASE 9)
What elements of the history would you ask about in order to formulate your diagnosis? Choose up to five.
1. Diffuse muscle cramps
2. Alcohol intake
3. Effect of NSAIDs
4. Family history of back problems
5. Previous history of back injury
6. Shoulder pain
7. Weakness in right leg
8. Allergies
9. Knee problems
10. Urinary incontinence
11. Skin rashes
12. Previous effective physiotherapy
13. Headaches
14. Nocturia

36 Sample Questions

Key Feature Scoring Guide

Toronto Notes 2005

Key Feature Scoring Guide


QUESTION 5, CASE 2

QUESTION 1, CASE 1
1.

Maximum Number to be selected:

1.

Maximum Number to be selected:

2.

The Number you selected (S):

___

2.

The Number you selected (S):

___

3.

Is S greater than the Maximum?

Your Score = 0.
Continue.

3.

Is S greater than the Maximum?

4.

There are 5 correct answers:

2, 4, 8, 13, 20

4.

There are 4 correct answers:

6, 13, 16, 19

5.

Number of correct answers you selected:

5.

Number of correct answers you selected:

___

YES
NO

___

YES
NO

Your Score = 0.
Continue.

QUESTION 6, CASE 3

QUESTION 2, CASE 1
1.

Maximum Number to be selected:

1.

Maximum Number to be listed:

2.

The Number you selected (S):

___

2.

The Number you listed (L):

___

3.

Is S greater than the Maximum?

Your Score = 0.
Continue.

3.

Is L greater than the Maximum?

4.

There are 5 correct answers:

9, 13, 14, 18, 20

4.

There are 2 correct answers:

5.

Number of correct answers you selected:

YES
NO

YES
NO

Your Score = 0.
Continue.

1. Delirium
2. Acute Confusional State

___
5.

Number of correct answers you listed:

___

QUESTION 3, CASE 2
1.

Maximum Number to be listed:

2.

The Number you listed (L):

___

3.

Is L greater than the Maximum?

4.

There are 8 correct answers:


1.
2.
3.
4.
5.
6.
7.
8.

5.

YES
NO

Your Score = 0.
Continue.

Prematurity
Difficult child
History of parental child abuse
History of substance abuse
Social isolation
Developmental delay
Poverty
Parental character/personality disorder

Number of correct answers you listed:

1.

Maximum Number to be selected:

2.

The Number you selected (S):

___

3.

Is S greater than the Maximum?

4.

There are 5 correct answers:

1, 5, 12, 14, 15

5.

Number of correct answers you selected:

___

YES
NO

Your Score = 0.
Continue.

QUESTION 8, CASE 3
=

___

QUESTION 4, CASE 2
1.

Maximum Number to be selected:

2.

The Number you selected (S):

___

3.

Is S greater than the Maximum?

4.

There are 4 correct answers:

7, 9, 14, 19

5.

Number of correct answers you selected:

YES
NO

QUESTION 7, CASE 3

Your Score = 0.
Continue.

___

1.

Maximum Number to be selected:

2.

The Number you selected (S):

___

3.

Is S greater than the Maximum?

4.

There are 5 correct answers:

3, 5, 6, 11, 19

5.

Number of correct answers you selected:

YES
NO

Your Score = 0.
Continue.

___

Toronto Notes 2005

Key Feature Scoring Guide

QUESTION 9, CASE 4

Sample Questions 37

QUESTION 13, CASE 5

1.

Maximum Number to be selected:

1.

Maximum Number to be selected:

2.

The Number you selected (S):

___

2.

The Number you selected (S):

___

3.

Is S greater than the Maximum?

Your Score = 0.
Continue.

3.

Is S greater than the Maximum?

4.

There are 4 correct answers:

1, 4, 6, 8

4.

There are 8 correct answers:

5.

Number of correct answers you selected:

___

2, 3, 5, 6, 7, 12,
13, 15

5.

Number of correct answers you selected:

___

YES
NO

QUESTION 10, CASE 4

YES
NO

Your Score = 0.
Continue.

QUESTION 14, CASE 5

1.

Maximum Number to be selected:

1.

Maximum Number to be selected:

2.

The Number you selected (S):

___

2.

The Number you selected (S):

___

3.

Is S greater than the Maximum?

Your Score = 0.
Continue.

3.

Is S greater than the Maximum?

4.

There are 8 correct answers:

1, 2, 4, 5, 7, 8,
10, 14

4.

There are 4 correct answers:

2, 6, 7, 10

5.

Number of correct answers you selected:

5.

Number of correct answers you listed:

___

YES
NO

___

YES
NO

Your Score = 0.
Continue.

QUESTION 15, CASE 6


QUESTION 11, CASE 4

1.

Maximum Number to be listed:

1.

Maximum Number to be selected:

2.

The Number you listed (L):

___

2.

The Number you selected (S):

___

3.

Is L greater than the Maximum?

3.

Is S greater than the Maximum?

Your Score = 0.
Continue.

4.

There are 3 correct answers:

4.

There are 7 correct answers:

3, 5, 7, 8, 9, 13, 18

5.

Number of correct answers you selected:

YES
NO

YES
NO

Your Score = 0.
Continue.

1. Biliary colic
2. Acute cholecystitis
3. Bile duct carcinoma

___
5.

Number of correct answers you listed:

___

QUESTION 12, CASE 5


1.

Maximum number to be selected

QUESTION 16, CASE 6

2.

The number you listed (L):

___

1.

Maximum Number to be selected:

3.

Is L greater than the Maximum?

2.

The Number you selected (S):

___

3.

Is S greater than the Maximum?

4.

There are 4 possible answers:


4.

There are 7 correct answers:

1, 2, 3, 5, 6, 7, 8

5.

Number of correct answers you selected:

YES
NO

Your Score = 0.
Continue.

1. Bacterial vaginosis
2. Vaginal/vulvar candidiasis
3. Trichomonas vaginalis
4. Chemical vaginitis
5.

Number of correct answers you listed:

___

YES
NO

Your Score = 0.
Continue.

___

38 Sample Questions

Key Feature Scoring Guide

QUESTION 17, CASE 7

Toronto Notes 2005

QUESTION 20, CASE 8

1.

Maximum Number to be listed:

1.

Maximum Number to be selected:

2.

The Number you listed (L):

___

2.

The Number you selected (S):

___

3.

Is L greater than the Maximum?

Your Score = 0.
Continue.

3.

Is S greater than the Maximum?

4.

There are 5 correct answers:

4.

There are 4 correct answers:

1, 6, 8, 9

1.
2.
3.
4.
5.

5.

Number of correct answers you selected:

___

5.

YES
NO

Hemorrhoids
Anal fissure
Colon or rectal cancer
Proctitis or IBD or colitis
Diverticulosis

YES
NO

Your Score = 0.
Continue.

QUESTION 21, CASE 9

Number of correct answers you listed:

___

QUESTION 18, CASE 7

1.

Maximum Number to be selected:

2.

The Number you selected (S):

___

3.

Is S greater than the Maximum?

4.

Essential answers are:

7 and 10

5.

Did you select both essential answers?


NO
YES

Your Score = 0.
Continue.

YES
NO

Your Score = 0.
Continue.

1.

Maximum Number to be selected:

2.

The Number you selected (S):

___

3.

Is S greater than the Maximum?

4.

There are 6 correct answers:

1, 5, 8, 9, 10, 13

6.

There are 5 correct answers:

3, 5, 7, 10, 12

5.

Number of correct answers you selected:

7.

Number of correct answers you listed:

___

YES
NO

Your Score = 0.
Continue.

___

QUESTION 19, CASE 8

QUESTION 22, CASE 9

1.

Maximum Number to be listed:

1.

Maximum Number to be selected:

2.

The Number you listed (L):

___

2.

The Number you selected (S):

___

3.

Is L greater than the Maximum?

Your Score = 0.
Continue.

3.

Is S greater than the Maximum?

4.

There are 4 correct answers:

4.

There are 8 correct answers:

1.
2.
3.
4.

2, 4, 7, 8, 10, 11,
14, 16

5.

Number of correct answers you selected:

5.

YES
NO

Transient tachypnea of the newborn or wet lung


Meconium aspiration
Pneumonia
Cardiac disease

Number of correct answers you listed:

___

YES
NO

Your Score = 0.
Continue.

___

You might also like